You are on page 1of 79

Anesthesia/Physics MCQs with Model Answers

"FRCA"
(http://www.frca.co.uk/SectionContents.aspx?
sectionid=75)

MCQ A
Created: 29/9/2005
 

1. Intermittent porphyria:

a) Is autosomal recessive
b) Can present with autonomic dysfunction
c) Can present with abdominal pain and no other signs
d) Can present with psychosis
e) Can be started by ethanol

2. A Nitrogen washout curve is used to measure:

a) Anatomical dead space


b) Physiological dead space
c) Vital capacity
d) Uneven ventilation of different areas of lung
e) Diffusion capacity

3. Amitryptiline overdose is commonly associated with:

a) Convulsions
b) Dysrhythmias
c) Metabolic acidosis
d) Hypothermia
e) Hypercapnia

4. The 'bleeding time' is increased in:

a) Haemophilia
b) Idiopathic thrombocytopenic purpura
c) Coumarin overdose
d) Vitamin C deficiency

5. Regarding the Severinghaus CO2 electrode:

a) The surrounding medium is bicarbonate


b) It contains CO2-sensitive glass
c) It is more accurate with gases than blood

-1-
d) It is temperature sensitive

6. The following methods can block sensory input from the larynx:

a) LA spray to the cords


b) LA to the pyriform fossa
c) Retrograde LA spray through cricothyroid membrane
d) Superior laryngeal nerve block in hypothyroid groove
e) Recurrent laryngeal nerve block

7. Pulmonary artery wedge pressure can be increased:

a) After myocardial infarction


b) In mitral stenosis
c) In aortic incompetence
d) In pulmonary fibrosis
e) In pulmonary stenosis

8. In the myasthenic syndrome there is:

a) Sensitivity to depolarising muscle relaxants


b) Sensitivity to non-depolarising muscle relaxants
c) Post-tetanic potentiation
d) Improvement with repeated muscle activity
e) Decreased voltage on the EMG

9. Paracetamol:

a) Is fully absorbed from the stomach


b) Is an aniline derivative
c) Is converted to phenacetin
d) Is a useful anti-inflammatory agent
e) Is excreted unmetabolised

10. In phaeochromocytoma treated with beta blockers, there is:

a) Decreased systolic blood pressure


b) Decreased heart rate
c) Miosis
d) Cold peripheries
e) Postural hypotension

11. Dystrophia myotonica is associated with:

a) Cataracts
b) Frontal baldness
c) Temporalis wasting
d) Sternomastoid wasting
e) Dysarthria
f) Sensitivity to non-depolarising muscle blockers

-2-
12. The following statements concerning pulmonary function are true:

a) Low forced expiratory volume/forced vital capacity occurs in restrictive conditions


b) Peak expiratory flow rate can be measured using a pneumotachograph
c) Peak expiratory flow rate is unrelated to effort
d) A vitalograph may be used to measure the peak expiratory flow rate
e) In obstructive conditions, the forced vital capacity is reduced

13. Halothane concentrations can be measured using:

a) Absorption on to silicone rubber


b) Ultraviolet light absorption
c) Infrared light absorption
d) Refractrometry
e) Paramagnetism

14. The standard deviation of a sample taken from a normally distributed


population:

a) Is the square root of variance


b) Is greater than the standard error
c) Indicates the limits from the mean that contain 2/3 of the values
d) Reflects the bias in selecting the original population
e) Is greater than the mean

15. A Wright's respirometer:

a) Measures volume directly


b) Gives falsely low readings at high flow rates
c) Has an internal volume of 150 ml
d) May be used to measure peak flow rate
e) Is unaffected by the composition of the gases being measured

16. The following endobronchial tubes have no carinal hook:

a) Carlens
b) Robertshaw
c) McIntosh-Leatherdale
d) Bryce-Smith
e) White

17. In a patient with jaundice, the following suggest obstruction:

a) No urobilinogen in urine
b) Increased stercobilinogen
c) Raised aspartate transaminase
d) Normal acid phosphatase
e) Normal alkaline phosphatase
f) Increased concentration bilirubin in urine therefore dark

-3-
g) Coagulopathy

18. Creatinine – phosphokinase is raised:

a) 24 hours post-myocardial infarction


b) In muscular dystrophy
c) In myasthenia gravis
d) In thyrotoxicosis
e) In osteomalacia

19. The recurrent laryngeal nerve:

a) Supplies some intrinsic muscles of the larynx


b) Supplies the cricothyroid muscle
c) Supplies the inferior constrictor muscle
d) Supplies sensation below the vocal cords
e) Supplies sensation to the posterior 2/3 of the tongue

20. Regarding the "Fluotec Mark III" vaporiser:

a) The temperature control is a bellows mechanism


b) The termperature control is at the vaporising chamber outlet
c) When filled with isoflurane it will deliver approximately the dialled concentration
d) It is not accurate at flows of less than 2 litres/minute

21. Mydriasis can occur in:

a) Horner's syndrome
b) Neurosyphilis
c) Untreated organophosphate poisoning
d) Oculomotor palsy

22. Prolonged irrigation of the bladder with isotonic saline for transurethral
resection TUR is associated with:

a) Hyponatraemia
b) Hypernatraemia
c) Hypokalaemia
d) Reduced osmolarity
e) Haemolysis
f) Confusion

23. In a patient who has had a traumatic quadriplegia for one week:

a) Intermittent positive pressure ventilation can cause hypotension


b) Suxamethonium sensitivity occurs
c) Retention of urine occurs
d) Increased tendon jerks occur
e) Steroids improve prognosis

-4-
ANSWERS

1.FTTTT
2.TFFTF
3.TTTFT
4.FTFT
5.TFFT
6.TTTTT
7.TTTTF
8.TTTTF
9.FTFFF
10.TFFFT
11.TTTTFT
12.FTFFT
13.TTTTF
14.TTTFF
15.FFFFT
16.FTTTF
17.TFFFFTT
18.TTFFF
19.TFFTF
20.FFTF
21.FFFT
22.TFTFFT
23.TTTTF

MCQ B
Created: 29/6/2004
Updated: 5/10/2006
  1. Ritrodrine can cause:

a) bradycardia
b) heart block
c) Left ventricular failure
d) hypotension
e) peripheral vasoconstriction

2. Trigeminal ganglion block causes ipsilateral analgesia of:

a) lower lip
b) inside the nose
c) angle of the jaw
d) external auditory meatus
e) soft palate

3. A pressure cycled ventilator:

-5-
a) is a minute volume divider
b) is cycled from inspiration to expiration after a set time
c) is cycled from expiration to inspiration after a set time
d) has end expiratory pressure determined by the patient's compliance
e) ventilates a patient with obstructive airways disease more efficiently if operated at
low flows

4. Airways resistance:

a) is greater during inspiration than expiration


b) can be measured by whole body plethysmography
c) is increased by ephedrine
d) is one factor determining the work of breathing
e) is expressed in kPa/L/s

5. Regarding Mapleson's breathing systems:

a) 'D' is identical to Magill


b) some degree of rebreathing can occur with all systems
c) 'A' requires the same for spontaneous ventilation and IPPV
d) 'F' system is used in paediatrics
e) 'E' system is the most efficient for spontaneous ventilation

6. Recognised effects of positive end-expiratory pressure include:

a) sodium retention
b) fall in cardiac output
c) rise in closing volume
d) rise in functional residual capacity
e) fall in central venous pressure

7. An elderly man given atropine becomes excited and confused; appropriate


treatment includes:

a) morphine
b) physostigmine
c) intubation and ventilation
d) droperidol
e) chlorpromazine

8. Causes of increased bleeding during head and neck surgery include:

a) hypoxia
b) hypocarbia
c) respiratory obstruction
d) chronic aspirin ingestion
e) transfusion of excessive quantities of stored blood

9. In cardioversion for dysrhythmias:

-6-
a) the shock is given on the upstroke of the T wave
b) general anaesthesia is always required for synchronised cardioversion
c) AC is safer than DC current
d) no preoperative assessment is needed for elective cases
e) ventricular fibrillation may result

10. Hyperventilation throughout general anaesthesia results in:

a) increased anaesthetic requirements


b) shift of the oxygen dissociation curve to the right
c) increased arterial to venous pO2 difference
d) reduced renal blood flow
e) hyperkalaemia

11. In patients with haemorrhagic shock:

a) physiological dead space is increased


b) renal blood flow is decreased
c) antidiuretic hormone secretion is increased
d) the oxygen dissociation curve is shifted to the left
e) O2 delivery is decreased

12. Regarding dental anaesthesia:

a) oral debris inhalation is unlikely if the patient is supine


b) intermittent methohexitone is a satisfactory technique for the operator anaesthetist
c) the number of dental general anaesthetics given is steadily increasing
d) demand flow anaesthetic systems result in economy of gases
e) simple monitoring is not essential

13. The right lung has:

a) one fissure
b) no Sibson's fascia
c) direct relationship to the azygos vein
d) two pulmonary veins
e) seven broncho-pulmonary segments

14. Six hours after thoracotomy for oesophageal resection:

a) vital capacity is reduced


b) functional residual capacity is reduced
c) peak flow is reduced
d) venous admixture is reduced
e) pO2 on air is reduced

15. Cardioplegia:

a) contains a high concentration of calcium ions

-7-
b) contains a high concentration of potassium ions
c) contains local anaesthetic
d) stops the heart in diastole
e) is given at 8 degrees C

16. The critical temperature of a gas is that:

a) at which the pressure is the critical pressure


b) at which freezing takes place
c) at which attraction between gas molecules is negligible
d) above which the gas cannot be ignited
e) at which Boyle's law is perfectly obeyed
f) above which it cannot be liquefied

17. Central venous pressure is raised and cardiac output is reduced in the
following:

a) Addison's disease
b) pulmonary embolism
c) air embolism
d) tension pneumothorax
e) pre-eclampsia

18. Hyaline membrane disease:

a) usually occurs within 12 hours of delivery


b) usually occurs after Caeserean section
c) has a better prognosis if steroids are given to the infant
d) is uncommon after 36 weeks' gestation
e) is more common in multiple pregnancies

19. Raised left ventricular end-diastolic pressure implies:

a) increased left ventricular compliance


b) decreased left ventricular left ventricular stroke work index
c) decreased oxygen flux
d) decreased myocardial oxygen demand
e) decreased myocardial oxygen supply

20. Internal carotid artery disease can cause:

a) transient loss of vision


b) tunnel vision
c) paraesthesiae on the side of the face
d) dysphagia
e) Parkinsons's disease

21. Pulmonary capillary wedge pressure is a reliable parameter:

a) in mitral stenosis

-8-
b) after anterior myocardial infarction
c) in pulmonary stenosis
d) in aortic stenosis
e) in pulmonary fibrosis

22. Vitamin B12 is appropriate treatment in:

a) vegans
b) pregnancy
c) Crohn's disease
d) partial gastrectomy
e) microcytic anaemia

23. The following are intimately related to the neck of a femoral hernia:

a) pubic tubercle
b) inguinal ligament
c) inferior epigastric artery
d) femoral vein
e) femoral artery

24. The following result from ulnar nerve block at the wrist:

a) weakness of adductor pollicis


b) paraesthesiae in the palmar aspect of the little finger
c) paraethesiae in the dorsal aspect of the little finger
d) vasodilatation in the nerve distribution
e) analgesia of C8 dermatome

25. The celiac plexus is related:

a) anteriorly to the crura of the diaphragm


b) anteriorly to the inferior vena cava
c) anteriorly to the aorta
d) to the L3 vertebra
e) posteriorly to the pancreas

26. In cases of gastrointestinal bleeding:

a) blood volume falls by 30% before a fall in blood pressure is observed


b) there is a higher incidence in males than females
c) the cause is more likely to be a duodenal than a gastric ulcer
d) tachycardia may present before overt blood loss
e) surgery must take place before resuscitation

27. Reflex sympathetic dystrophy:


a) may be associated with osteoporosis
b) may be associated with an increase in skin temperature
c) often presents with pain as the presenting complaint
d) is associated with vasomotor disturbances

-9-
e) is more common in athletes

28. For amputation at mid-thigh, the following nerves must be blocked:

a) obturator
b) femoral
c) sciatic
d) genitofemoral
e) lateral cutaneous nerve of the thigh

29. The after-coming twin becomes "stuck" following the administration of


ergometrine. The following drugs will reliably relax the uterus:

a) thiopentone
b) suxamethonium
c) ritodrine
d) salbutamol
e) halothane

30. Raised creatinine phosphokinase:

a) occurs after the administration of suxamethonium


b) implies a recent myocardial infarction
c) may reflect an increased risk of malignant hyperpyrexia
d) is an indication of alcoholic liver disease
e) is seen in acute pancreatitis

31. The following drugs may be safely used in porphyria:

a) barbiturates
b) bupivacaine
c) lidocaine
d) chlorpromazine
e) sulphonamides

32. An aortic aneurysm can be said to be leaking if there is:

a) loin pain
b) back pain
c) anuria
d) obstructed bowel sounds
e) no femoral pulse
f) absent bowel sounds

33. In septic shock:

a) peripheral hypothermia is associated with good prognosis


b) the patients usually have an increased cardiac output
c) the patients usually have a depleted circulating volume
d) the causative organisms are always Gram negative

- 10 -
e) antibiotics should not be given before blood culture results are available

34. Rheumatoid arthritis is associated with:

a) pericarditis
b) aortic stenosis
c) tricuspid incompetence
d) aortic aneurysm

35. After head injury, increased intracranial pressure is indicated by:

a) fall in blood pressure


b) reduction in the Glasgow coma score
c) increase in heart rate
d) increase in pCO2
e) small pupils

36. In haemophilia:

a) the mode of inheritance is X-linked


b) the thromboplastin generation test may be abnormal despite a normal bleeding time
c) factor VIII and IX clotting factor concentrates form an important treatment
modality
d) haematomas are treated by aspiration
e) disseminated intravascular coagulation  is a common complication following surgery

37. In the diagnosis of brain death:

a) lack of EEG activity is essential


b) caloric tests must be performed bilaterally
c) the admitting consultant must certify death
d) lack of stretch reflexes in all limbs is essential
e) the pupils must be fixed and dilated

38. Acute frontal sinusitis may cause:

a) subdural abscess
b) cerebral abscess
c) spreading osteomyelitis
d) orbital cellulitis
e) meningitis

39. In the ECG:

a) T wave indicates repolarisation


b) the transmembrane potential is the ratio of K+ in to K+ out
c) a single V lead requires one connection only
d) QRS width is proportional to muscle mass
e) negative deflection indicates movement away from the electrode

- 11 -
40. Diazoxide in pre-eclampsia:

a) reduces the diastolic blood pressure


b) relaxes the uterus
c) prevents hypoglycaemia
d) promotes diuresis
e) causes sedation

41. Vomiting is affected by:

a) age
b) weight
c) the choice of premedicant
d) intraoperative hypotension
e) the duration of anaesthesia

42. Dysphagia may be associated with:

a) cervical rib
b) pharyngeal pouch
c) Plummer-Vinson syndrome
d) achalasia
e) carcinoma of the bronchus

43. Carcinoma of the bronchus may be associated with:

a) pink palms
b) thirst
c) Horner's syndrome
d) dysphagia
e) cerebellar ataxia

44. An 8-year-old child is rescued 20 minutes after drowning and has a core
temperature of 30 degrees C and fixed dilated pupils. Further appropriate
treatment includes:

a) phenobarbitone
b) rapid rewarming
c) hypoventilation
d) steroids
e) cardiopulmonary resuscitation

45. The recurrent laryngeal nerve supplies:

a) sensation below the cords


b) the intrinsic muscles of the pharynx
c) the cricothyroid muscle
d) the inferior constrictor muscle
e) the epiglottis

- 12 -
ANSWERS
 
1.FFTTF 23.TTFTF
2.TTFFF 24.TTTTF
3.FFFFF 25.TFTFT
4.FTFTT 26.FTTTF
5.FTFTF 27.TTTTF
6.TTFTF 28.FTFFT
7.FTFFF 29.FFTTT
8.TFTTT 30.TTTFF
9.FFFFT 31.FTFTF
10.FFFTF 32.TTTFTT
11.TTTFT 33.FTTFF
12.FFFTF 34.TFFF
13.FFTTF 35.FTFTF
14.TTTFT 36.TTTFF
15.FTTTF 37.FTFFF
16.TFFFFT 38.TTFTT
17.FTTTF 39.TFFFT
18.FFFTT 40.TFTFF
19.FFTFT 41.TTTTF
20.TFTTT 42.FTTTT
21.FTTTF 43.FTTTT
22.TFTTF 44.FFFFT
  45.TFFFF

ArticleDate:20040629
Site

Sec

MCQ C
Created: 2/4/2006
  1. Regarding the Bain circuit:

a) it is suitable for small children


b) it can act as a Mapleson C circuit
c) inspiratory flow occurs through the outer tubing
d) a disconnection at the machine causes increased dead space

2. Regarding the Bain coaxial circuit:

a) it may function as a Mapleson C


b) it requires FGF=MV to prevent rebreathing in spontaneous respiration
c) it can be used in small children
d) dead space increases if the inner tube becomes detached at the machine end
e) FGF occurs through the outer tube

3. Regarding the Manley ventilator:

a) it is a minute volume divider


b) it is a pressure generator
c) it is time cycled
d) it is suitable for ventilating asthmatics
e) it leads to rotameter inaccuracy

- 13 -
4. The following tubes used in pulmonary surgery have a carinal hook:

a) Carlens
b) Gordon Green
c) McIntosh-Leatherdale
d) Bryce-Smith
e) Robertshaw
f) White

5. A stellate ganglion block causes:

a) pupil dilation
b) enopthalmos
c) anaesthesia of the superglottic part of the larynx
d) increased cerebral blood flow

6. A maxillary nerve block in the pterygopalatine fossa gives anaesthesia of:

a) upper molars
b) upper incisors
c) soft palate
d) anterior 2/3 of tongue
e) anterior part of nasal septum

7. Supraclavicular brachial plexus block differs from auxiliary block in that:

a) there is a greater risk of intravascular injection


b) analgesia of the shoulder is poor
c) it is more likely to give failure to block the interosseous nerves
d) it is less likely to give analgesia to the lateral forearm
e) there is an increased incidence of pneumothorax
f) it is less likely to give analgesia to the under side of the forearm

8. Intercostal block of one rib at the posterior angle causes:

a) somatic analgesia throughout the dermatome


b) visceral analgesia
c) low blood pressure because of preganglionic sympathetic block
d) increased blood levels of local anaesthetic
e) vasoconstriction

9. Hypotension during spinal anaesthesia may be due to:

a) preganglionic autonomic blockade


b) venoconstriction
c) block of the dorsal roots
d) ischaemia of the vasomotor centre
e) block of nerves to the adrenal medulla

- 14 -
10. The volume displacement of an electromaster:

a) indicates the amount of liquid needed to fill it


b) depends on the volume of the chamber
c) depends on the stiffness of the diaphragm
d) depends on the frequency response

11. Halothane concentrations can be measured by:

a) ultraviolet absorption
b) infrared absorption
c) a refractometer
d) changes in the elasticity of silicone rubber

12. Paramagnetism is used in:

a) O2 analysis
b) CO2 analysis
c) halothane vapour analysis

13. Radiation is the emission of:

a) beta particles
b) alpha particles
c) neutrons
d) rays
e) any wave form

14. Regarding gas chromatography:

a) it uses carrier gas


b) it uses electron deflection
c) it uses the Venturi principle
d) it uses haemodilution
e) it uses transillumination
f) a flame ionisation detector, katharometer, electron capture detector are required

15. Pressure gauges:

a) work on the Bourdon principle


b) can be used to regulate gas flow
c) can convert gas at a high pressure into gas at a low pressure
d) form part of a flow meter

16. Success of cricoid pressure in preventing aspiration depends on:

a) absence of a nasogastric tube


b) an intact cricoid cartilage
c) the oesophagus being pressed onto the vertebral body

- 15 -
d) extension of the neck
e) preoxygenation for 5 minutes

17. After massive inhalation of gastric acid, one would expect the following:

a) lung abscesses
b) severe hypercapnoea
c) bacteraemia
d) hypovolaemia
e) destruction of surfactant

18. In prolonged operations, core temperature is accurately monitored at:

a) the tympanic membrane


b) the lower third of the oesophagus
c) the muscle in the thigh
d) the nasopharynx
e) the rectum

19. The following may contribute to the development of a postoperative cauda


equina syndrome:

a) age
b) the use of adrenalin in epidurals
c) Trendelenburg position
d) spinal barbotage

20. During anaesthesia, a patient with Parkinson's disease on levodopa should


not receive:

a) enflurane
b) fentanyl
c) morphine
d) droperidol
e) nitrous oxide

21. In the fetal transfer of drugs given by the epidural route:

a) the placenta is an effective barrier


b) foetal bradycardia may occur with local analgesics
c) highly protein-bound drugs are transferred less
d) amide-linked local analgesics should be avoided

22. For Caesarean section under epidural anaesthesia:

a) never give more than 20 ml of 0.5% bupivacaine


b) the block must extend from T5 to S1
c) syntocinon, rather than ergometrine, must be given
d) a paracervical block must also be performed

- 16 -
e) it is necessary to preload with 500 ml Hartmann's solution

23. Features of amniotic fluid embolus include:

a) convulsions
b) cardiovascular collapse
c) bronchospasm
d) abnormal bleeding
e) pulmonary hypertension

24. Block of the trigeminal ganglion is associated with anaesthesia of:

a) alar nasi
b) lower lip
c) angle of mandible
d) soft palate
e) eardrum

25. If 50% nitrous oxide is inhaled for 3 days:

a) the lymphocyte count falls


b) methionine synthetase activity is reduced
c) megaloblastic bone marrow changes occur
d) vitamin B12 deficiency anaemia develops
e) peripheral neuropathy develops

26. Helium:

a) is less viscous than O2


b) is in liquid form in brown cylinders
c) inhalation causes voice changes
d) is used to decrease the work of breathing in bronchospasm
e) supports combustion

27. Methohexitone:

a) has a pH less than 8 in a 1% solution


b) is a methylated thiobarbiturate
c) has a longer duration of action than thiopentone
d) does not cause pain on injection
e) is associated with fewer excitatory phenomena than thiopentone

28. Isoflurane:

a) has a similar boiling point to enflurane


b) has a similar saturated vapour pressure to halothane
c) has a lower blood/gas coefficient than halothane
d) is extensively metabolised
e) has a minimum alveolar concentration of 1.68

- 17 -
29. Nitroprusside reduces blood pressure by:

a) acting on alpha-receptors
b) reducing the cardiac output
c) causing a bradycardia
d) producing cyanide ions
e) a direct action on blood vessels

30. The following are chiefly eliminated by the kidney:

a) d-tubocurarine
b) suxamethonium
c) dopamine
d) gallamine
e) digoxin

31. Edrophonium has the following clinical uses:

a) as a test for myasthenia gravis


b) for the reversal of neuromuscular blockade
c) as a centrally acting respiratory stimulant
d) for the relief of urinary retention
e) for the treatment of myotonia congenita

32. An untoward reaction to the following may occur in a patient on monoamine


oxidase inhibitors:

a) adrenaline
b) thiopentone
c) diazepam
d) tricyclic antidepressants
e) amphetamine

33. A 60-year-old man develops painless jaundice, pale stools and dark urine;
which of the following are true:

a) this is compatible with a raised alkaline phosphatase


b) the urinary urobilinogen will be raised
c) an oral cholecystogram will be useful
d) the absence of pain excludes malignancy
e) itching may be present

34. A low serum potassium is associated with:

a) Cushing's disease
b) hyperventilation
c) carbenoxolone therapy
d) triamterene therapy

- 18 -
e) metabolic alkalosis

35. Hypokalaemia requiring treatment is found in the following situations:

a) following cardiac bypass (open heart surgery)


b) during treatment for diabetic ketoacidosis
c) after major burns
d) after giving depolarising relaxants

36. The following are causes of a metabolic acidosis:

a) uretero-colic fistula
b) vomiting
c) diarrhoea
d) CO2 retention

37. Hypoalbuminaemia:

a) may lead to hypocalcaemia


b) is a manifestation of hypopituitarism
c) can cause tetany
d) can be a feature of carcinoma of the sigmoid colon
e) may develop in hepatic cirrhosis

 ANSWERS

1.FFFT 24.TTFTT
2.FFFTF 25.TTTTT
3.TTTFT 26.FFTFF
4.TTFFFT 27.FFFFF
5.FTFT 28.FTTFF
6.TTFFT 29.FFFFT
7.FFFFTT 30.TFFTT
8.TFFTF 31.TTFFF
9.TFFFF 32.FFFTT
10.FFTF 33.TFFFT
11.TTTT 34.TTTFT
12.TFF 35.TTFF
13.TTTTF 36.TFTF
14.TFFFFT 37.TFFTT
15.TFFF
16.TTTTF
17.FFFTT
18.TTFFF
19.TTFF
20.FFFTF
21.FTTF
22.FFFFF
23.TTTTT
 

ArticleDate:20060402
SiteSect

- 19 -
Created: 2/4/2006
MCQ D
  1. Concerning the history of anaesthesia:

a) Thomas Morton gave the first public demonstration of ether


b) Nieman was the first to isolate cocaine
c) Waters was the first anaesthetist to use halothane
d) Humphrey Davey isolated oxygen
e) Snow gave ether to Queen Victoria

2. Metabolic alkalosis with hypokalaemia is caused by:

a) frusemide
b) Cushing's syndrome
c) Addison's disease
d) vomiting

3. Digoxin toxicity may cause:

a) supraventricular tachycardia
b) ventricular ectopics
c) liver damage
d) atrioventricular block
e) jaundice

4. In a circle anaesthetic system:

a) the soda lime canister should be equal in volume to the patient's tidal volume
b) if a vaporiser is put in the circle, it should be placed between the inspiratory limb
and the fresh gas flow inlet
c) the volume of the reservoir bag is not critical
d) the adjustable pressure-limiting valve should be between the expiratory limb and the
soda lime canister

5. Signs of fat embolism include:

a) pyrexia
b) bradycardia
c) petechial rash
d) bronchospasm
e) mental changes

6. The following can act by competitive antagonism:

a) naltrexone
b) nalorphine
c) neostigmine
d) flumazenil
e) nalbuphine

- 20 -
7. During cardiopulmonary resuscitation, the following are indicated:

a) lidocaine as first pharmacological therapy for ventricular fibrillation


b) for ventricular fibrillation, DC shock should start at 50 joules
c) calcium should not be given intravenously to a patient having haemodialysis
d) bicarbonate should be given every 10 minutes
e) adrenaline should be given in a dose of 0.5 mg via the endotracheal tube

8. Regarding damage to the vestibular apparatus caused by gentamicin:

a) it is irreversible
b) it does not occur with oral gentamicin
c) calorie testing is normal

9. The laryngeal mask may be useful for:

a) ventilation
b) resuscitation
c) intermittent positive pressure ventilation
d) microlaryngoscopy
e) ophthalmic anaesthesia

10. During a long operation, reliable monitors of core temperature include:

a) temperature probe at the tympanic membrane


b) temperature probe in the rectum
c) quadriceps muscle temperature
d) temperature probe in the nasopharynx
e) temperature probe in the oesophagus at the level of the cricoid

11. Vitamin B12 injections are appropriate for anaemia in:

a) vegans
b) Crohn's disease
c) phenytoin therapy
d) alcoholism
e) post-gastrectomy

12. Following head injury, signs which suggest the need for urgent craniotomy
include:

a) reduced conscious level


b) dilated pupil
c) hypotension
d) convulsions
e) cerebrospianl fluid rhinorrhea (persistent)
f) depressed # skull

13. A patient with paraplegia of recent onset with injury at T4 may have:

- 21 -
a) hypotension on intermittent positive pressure ventilation
b) adductor spasm
c) bradycardia
d) hypothermia
e) urinary retention

14. Side-effects of hiatus hernia are:

a) iron deficiency anaemia


b) aspiration
c) oesophageal stricture
d) Mallory-Weiss tear
e) gastric carcinoma

15. A large atrioventricular shunt causes:

a) increased cardiac output


b) increased total systemic vascular resistance
c) cold extremities
d) tachycardia
e) heart failure

16. Sympathectomy may be used in the treatment of:

a) Renaud's disesase
b) causalgia
c) hyperhydrosis
d) venous ulcers
e) pain of intermittent claudication

17. Methods of anaesthetising the larynx include:

a) topical anaesthesia to the cords


b) anaesthesia of the superior laryngeal nerve through the thyrohyoid membrane
c) local anaesthesia injected through the cricothyroid membrane
d) local anaesthesia applied to the pyriform fossa
e) recurrent laryngeal nerve block

18. Breathing 50% nitrous oxide in oxygen for 3 days causes:

a) megaloblasts in bone marrow


b) megaloblastic anaemia
c) peripheral neuropathy
d) inhibition of vitamin B12
e) reduced number of lymphocytes

19. Cyanosis at birth occurs in:

a) Fallot's tetralogy
b) transposition of the great vessels

- 22 -
c) pulmonary stenosis (isolated)
d) patent ductus arteriosus
e) ventricular septal defect

20. In aortic regurgition:

a) there is a pressure gradient across the valve


b) angina is only a feature if it is associated with ischaemic heart disease
c) the murmur is best heard at the apex, with the patient leaning to the left
d) pistol shots may be heard over the femoral arteries

21. Methods of rapidly lowering serum potassium include:

a) glucose and insulin


b) calcium gluconate
c) frusemide
d) bicarbonate
e) calcium resonium resin

22. A man collapses 72 hours after a total gastrectomy. The following


measurements are made:
Temperature, 39 degrees C
Blood pressure, 80/30 mmHg
Pulse, 110 bpm
Central venous pressure, +2 mmHg

The diagnoses may include:

a) septicaemia
b) myocardial infarction
c) atelectasis
d) breakdown of anastomosis
e) haemorrhage
f) tamponade

23. Concerning streptokinase:

a) it should not be repeated within 3 months


b) it should not be given within 24 hours of an operation
c) it is a tissue plasminogen activator
d) it has been shown to be the most clinically useful thrombolytic agent

24. Regarding reading blood pressure by an automatic non-invasive method:

a) it may over-read at high pressures


b) it may over-read at low pressures
c) it is affected by arrhythmias
d) it may cause ulnar nerve damage
e) the width of the cuff should be half the circumference of the arm

- 23 -
25. Treatment of overdose with amitriptyline may include:

a) beta-blockade
b) digitalisaion
c) isoprenaline infusion
d) atropine

26. Penicillins:

a) are all inactivated by penicillinase


b) are bacteriostatic
c) at on cell wall synthesis

27. The following may attenuate the pressor response to intubation:

a) enalapril
b) intravenous lidocaine
c) topical lidocaine to the cords
d) a large dose of induction agent
e) beta-blockade

28. The following cross the first rib:

a) the vagus nerve


b) the subclavian artery
c) the supratentorial membrane
d) the T1 nerve root
e) the sympathetic chain

29. Concerning an interscalene approach to the brachial plexus block:

a) it is at C6
b) the interscalene groove is accentuated by taking a deep breath
c) pneumothorax is rare
d) a total spinal may result

30. Midazolam is:

a) metabolised to active metabolites


b) acetylated in the liver
c) antagonised by flumazenil
d) able to produce anterograde amnesia
e) water soluble

31. Concerning coagulation:

a) continued bleeding from a venepuncture site suggests haemophilia


b) in von Willebrand's disease, the platelet count is normal
c) prothrombin time is normal in factor 5 deficiency
d) prothrombin time is normal in factor 8 deficiency

- 24 -
e) The reptilase time is abnormal if a patient is heparinised

32. Concerning stellate ganglion block:

a) oesophageal perforation may occur


b) diaphragmatic paralysis may result
c) Horner's syndrome is essential for a successful block
d) a successful block abolishes a change in skin resistance on stimulation
e) a vasovagal reaction can occur while performing it

33. The following must be blocked for mid-thigh amputation:

a) femoral nerve
b) lateral cutaneous nerve of the thigh
c) oburator nerve
d) sciatic nerve
e) genitofemoral nerve
f) both femoral and sciatic nerves
g) post-cutaneous nerve

34. Concerning pressure cycled ventilators:

a) end-inspiratory pressure is determined by the patient's compliance


b) they are usually cycled from inspiration to expiration by time
c) they may be cycled from expiration to inspiration by time
d) they are more useful for patients with airways obstruction if operated at low flows
e) they are minute volume dividers

35. Concerning airway pressures:

a) peak and plateau pressures increase in patients with pneumothorax


b) they are high in obstructive airways disease

36. In the elderly:

a) chest wall compliance is decreased


b) vital capacity is decreased by 20 ml each year
c) closing volume is less than functional residual capacity
d) PaO2 is lower than in the young
e) in a 70-year-old, the alveolar/arterial oxygen difference is about 2.7 kPa

37. In a patient of 2 years old:

a) fluid replacement is 100 ml/kg/day


b) lung compliance is lower than in an adult
c) the narrowest point of the airway is the cricoid cartilage
d) blood volume is 50 ml/kg

38. A solitary nodule in the thyroid may be:

- 25 -
a) a thyroglossal cyst
b) Hashimoto's thyroiditis
c) the larges nodule of a multinodular goitre
d) a physiological goitre
e) a colloid goitre

39. The following may be useful indications in posterior fossa surgery:

a) a change in respiratory rate


b) arrhythmias
c) hypertension
d) delta waves on the EEG
e) a mill-wheel murmur

40. Desmopressin acetate may be:

a) useful for bleeding post-cardiopulmonary bypass


b) used to treat diabetes insipidus
c) used in primary noctural enuresis
d) used to treat bleeding peptic ulcers

ANSWERS

1.TTFFF 24.FTTTT
2.TTFT 25.TFFF
3.TTFTF 26.FFT
4.TFTFT 27.TTTTT
5.TFTTT 28.FTFTT
6.TTFTT 29.TTTT
7.FFFFF 30.FFTTT
8.TTF 31.FTFTF
9.TTTFT 32.TTTTT
10.TFFFF 33.TTFFFFT
11.TTFTT 34.FTTFF
12.FTFFTT 35.TT
13.TFTTT 36.TTFTT
14.TTTFF 37.TTTF
15.TFTTT 38.TFTFF
16.TTTFT 39.TTTFF
17.TTTTT 40.TTTF
18.TTTTT  
19.TTFTF
20.FFFT
21.TFFFF
22.TFFTFF
23.TTTF
 

MCQ E
Created: 29/6/2004
  1. Right lower lobe collapse is characterised by:

a) increased A-a difference


b) tachypnoea

- 26 -
c) increased PCO2
d) decreased arterial pH
e) an area of stony dullness

2. A haemoglobin of 8 g/dl and a reticulocyte count of 10% may occur in:

a) aplastic anaemia
b) haemolytic anaemia
c) iron deficiency anaemia
d) pernicious anaemia
e) acute leukaemia

3. Hypokalaemia may occur with:

a) spironolactone treatment
b) IV digoxin
c) compound sodium lactate infusion
d) metabolic acidosis
e) intermittent positive pressure ventilation with hypocapnia

4. In gout:

a) an acute attack may be precipitated by surgery


b) erythrocyte sedimentation rate is normal in an acute attack
c) urate deposits occur in the kidney
d) cystic lesions on X-rays are urate-filled deposits
e) family history occurs in less than 25% of patients with the primary variety

5. Complications of propanolol treatment are:

a) congestive cardiac failure


b) bronchospasm
c) retinal degeneration
d) hyperglycaemia
e) tachycardia

6. Signs of digoxin toxicity may occur with:

a) hyperkalaemia
b) hypercalcaemia
c) hypomagnesaemia
d) acute respiratory alkalosis
e) mannitol infusion

7. In the cardiovascular system:

a) sudden atrial fibrillation in a patient with mitral stenosis may result in pulmonary
oedema
b) dilatation of the right lower lobe veins is a reliable sign of pulmonary hypertension
c) tricuspid regurgitation is commonly of rheumatic origin

- 27 -
d) cachexia is a common feature of cardiac failure
e) cardiac failure due to aortic stenosis maybe irreversible due to myocardial fibrosis

8. Preoperative pacing is required in:

a) Wolf-Parkinson-White syndrome
b) first-degree heart block
c) type two second degree heart block
d) third-degree heart block
e) cardiogenic shock with sinus rhythm

9. ECG interference:

a) is reduced by screening of the leads


b) is reduced by main frequency of 100 Hz
c) is reduced by differential amplifiers
d) is reduced by decreased skin resistance
e) is reduced by the use of filters

10. Isolated pulmonary stenosis is associated with:

a) arterial cyanosis
b) parasternal leave
c) increased "A" wave in the jugular venous pressure
d) a loud pulmonary segment of the second heart sound
e) a systolic murmur and thrill at the left upper sternal edge

11. A full nitrous oxide cyclinder:

a) has a pressure of 5100 kPa at 200 degrees C


b) has a filling ratio of 0.67
c) should be inverted six times before connection to an anaesthetic machine
d) gives a constant pressure during usage
e) has its contents estimated by weight

12. In early sepsis syndrome:

a) cardiac output is normal


b) a decreased white cell count is a poor prognostic sign
c) adrenocorticotrophic hormone levels are low
d) Insulin is raised
e) PaO2 is lowered

13. Features of disseminated intravascular coagulation include:

a) a lowered fibrinogen level


b) a normal prothrombin time and activated partial thromboplastin time
c) heparin is a reasonable treatment following placental abruption
d) it may occur secondary to malaria

- 28 -
e) it may have a compensated phase with no bleeding

14. The following are features of barbiturate overdosage:

a) bullous skin reaction


b) myocardial depression
c) PaCO2 greater than kPa breathing spontaneously
d) aspiration of gastric contents
e) hypothermia

15. In a child who has been under water for 20 minutes and is unconscious with
fixed dilated pupils, immediate management should include:

a) barbiturate infusion
b) rapid rewarming
c) steroid therapy
d) hypoventilation
e) cardiac massage

16. The saturated vapour pressure of water:

a) is 0 at 273 K
b) is dependent on altitude
c) is barometric pressure when at boiling point
d) at 37 degrees C is the same as in blood

17. Supraclavicular block with respect to axillary block:

a) is more likely to give blockade of the lateral aspect of the forearm


b) gives better blockade of the shoulder
c) has a higher incidence of intravascular injection
d) gives a better block of muscles of the hand
e) has a higher incidence of pneumorthorax

18. The following are good indicators of the severity of an acute severe asthma
attack:

a) a low PaO2
b) a high PaO2
c) scattered rales and crackles
d) pyrexia
e) an increased pulsus paradoxus

19. Characteristics of chronic obstructive airways disease are:

a) a productive cough
b) right-sided heart failure is an early feature
c) abnormal arterial blood gases occur early
d) breathlessness is an early feature

- 29 -
20. Legionnaires' disease is associated with:

a) pneumonia
b) encephalopathy
c) a relative neutropenia
d) liver disease
e) conjunctivitis

21. Immediate signs of a massive pulmonary embolus include:

a) ischaemic cardiac pain


b) pleural effusion
c) lower lobe pulmonary venous congestion
d) peripheral cyanosis
e) haemoptysis

22. Recognised features of motor neurone disease are:

a) dysphagia
b) dysarthria
c) urinary retention
d) fasciculation
e) dementia

23. In Eisenmenger's syndrome:

a) hypovolaemia improves R to L shunt


b) hypoxia decreases R to L shunt
c) a ventricular septal defect is always present
d) a decrease in systemic vascular resistance reduces the R to L shunt
e) pulmonary hypertension is always present

24. Ways of assessing recovery used are:

a) Maddox Wing test


b) p deletion test
c) Romberg test
d) measurement of drug levels in blood
e) Stewart scoring system

25. The following effects can result from a stellate ganglion block:

a) miosis
b) exophthalmus
c) stuffy nose
d) dilatation of the cochlear artery
e) recurrent laryngeal nerve block

26. The following can result in pupillary dilatation:

- 30 -
a) trimetaphan
b) nitroprusside
c) stellate ganglion block
d) 0.6 mg of atropine IV
e) ecothiopate

27. A 3 in 1 block blocks:

a) ilioinguinal nerve
b) femoral nerve
c) genitofemoral nerve
d) obturator nerve
e) lateral femoral cutaneous nerve

28. Amniotic fluid embolism can cause:

a) bronchospasm
b) bleeding
c) peripheral cyanosis
d) pulmonary hypertension
e) hypertension

29. The birth canal is innervated by:

a) pudendal nerve
b) femoral nerve
c) obturator nerve
d) ilioinguinal nerve
e) genitofemoral nerve

30. A patient with infective endocarditis suddenly becomes dyspnoeic, with a


blood pressure of 130/50 mmHg. The venous pressure is raised and systolic and
diastolic murmurs become more pronounced. Which of the following may have
occurred:

a) rupture of an aortic cusp


b) sudden occurrence of a ventricular septal defect
c) inferior myocardial infarct
d) late reaction to the antibiotics
e) pulmonary embolism

31. Concerning hip arthroplasty:

a) methyl-methacrylate is a cardiac inotrope


b) hypoxia may be caused by marrow embolisation
c) regional techniques give rise to a greater overall survival rate
d) subcutaneous heparin will completely prevent deep vein thrombosis
e) hypocapnia produced intermittent positive pressure ventilation is beneficial to the patients

32. Concerning propofol:

- 31 -
a) it has no active metabolites
b) it can cause convulsions
c) it causes less nausea than thiopentone
d) it causes pain on injection more often than midazolam
e) it can cause metabolic alkalosis

33. Concerning alfentanyl:

a) it has a higher volume of distribution than fentanyl


b) it has a longer elimination half life than fentanyl
c) it is more potent than sufentanil
d) it is more protein bound than fentanyl
e) it can cause bradycardia

34. Concerning the minimum alveolar concentration of isoflurane:

a) it decreases with age


b) it is decreased with acute alcohol intake
c) it is lower in men than in women
d) it is higher in neonates compared with a two year old
e) it is decreased in pregnancy

35. Causes of prolonged postoperative unconsciousness:

a) acromegaly
b) an intracerebral event during the operation
c) myxoedema
d) prolonged action of muscle relaxants
e) hypoventilatoin

36. Concerning postoperative nausea and vomiting:

a) it is more common in women than in men


b) the incidence is 80% with general anaesthesia
c) it is more common with thiopentone that with propofol
d) butyrophenones can decrease the incidence
e) it is more common with ear surgery

37. Concerning day case surgery:

a) only ASA grade one patients are suitable


b) the operation should be done in such a way that no postoperative opioids are
needed
c) the patient should be accompanied home with an adult
d) a laparoscopic procedure is not suitable
e) intubation is not appropriate

38. Agents used to decrease the pressure response to intubation include:

- 32 -
a) ACE inhibitors
b) calcium antagonists
c) thiopentone
d) beta-blockers
e) fentanyl

39. The occurence of bronchospasm at the end of an operation to repair an


umbilical hernia in an asthmatic child can be caused by:

a) light anaesthesia
b) irritation of a tracheal tube
c) morphine in the premedication
d) isoflurane
e) reversal by neostigmine

40. During one-lung anaesthesia, the PaO2 is influenced by:

a) the amount of blood flow in the upper lung


b) the cardiac output
c) the mixed venous oxygen concentration
d) the haematocrit
e) the FIO2

41. In patients with pacemakers:

a) diathermy use should be avoided


b) hypovolaemia is poorly tolerated
c) electrolytes should be "normalised" prior to surgery
d) suxamethonium should be avoided
e) use of volatile agents can cause deterioration of function

42. Hyponatraemia may cause the following:

a) confusion
b) hypertension
c) convulsions
d) coma
e) tachycardia

43. A complete block of the ulnar nerve at the elbow will cause:

a) numbness on the ulnar side of the forearm


b) paralysis of the hypothenar muscles
c) paralysis of all thenar muscles
d) sensory loss over the little finger
e) sensory loss over the lateral side of the ring finger

44. Positive end-expiratory pressure can cause:

a) sodium retention

- 33 -
b) decreased cardiac output
c) increased residual volume
d) decreased functional residual capacity
e) decreased central venous pressure

45. The following features are essential to diagnose malignant hyperthermia:

a) muscle rigidity
b) hypercapnia
c) renal failure
d) body temperature greater than 38 degrees C
e) family history

46. Trigeminal nerve block is associated with anaesthesia of:

a) lower lip
b) angle of the mandible
c) soft palate
d) tip of the nose

47. Hypokalaemia can be caused by:

a) Addison's disease
b) major burns
c) alkalosis
d) triamterene therapy
e) diarrhoea

48. Gram-negative septicaemic shock is associated with:

a) urine output (0.5 ml/kg/h)


b) disseminated intravascular coagulation
c) hypotension unresponsive to fluid loading
d) high fever
e) diminished cardiac output

49. Laryngeal motor innervation is from the following:

a) glossopharyngeal nerve
b) internal laryngeal nerve
c) recurrent laryngeal nerve
d) hypoglossal nerve
e) superior laryngeal nerve

50. Uveitis is a feature of:

a) ulcerative colitis
b) rheumatoid arthritis
c) Crohn's disease
d) systemic lupus erythematosus

- 34 -
ANSWERS

1.TTFFF 24.TFFFT
2.FTFFF 25.TFTTT
3.FF?FT 26.TFFTF
4.TFTTF 27.FTFTT
5.TTFFF 28.TTTTF
6.FTTTF 29.TFFTT
7.TFFFT 30.TTFFF
8.FFTTF 31.FTFFF
9.TFTTT 32.TTTTF
10.FTTFT 33.FFFTT
11.FTFFT 34.TTFFT
12.TTFTT 35.TTTTT
13.TFFTT 36.TFTTT
14.TTTTT 37.FFTFF
15.FTFFT 38.TTTTT
16.TFTF 39.TTFFT
17.TTFFT 40.TTTTT
18.TTFFT 41.TTTTF
19.TFFF 42.T?TTF
20.TTFTF 43.FTFTF
21.TFFTF 44.TTTFF
22.TTFTT 45.FTFTF
23.TFFFT 46.TFFT
  47.FFTFT
48.TTTFF
49.FFTFT
50.FTTT

MCQ F
Created: 29/6/2004
  1. Mitral stenosis is associated with:

a) subacute bacterial endocarditis


b) rheumatic fever
c) pulmonary hypertension
d) systolic murmur at the apex

2. Parathyroid adenoma may present with:

a) psychiatric disturbance
b) increased serum calcium
c) decreased urinary calcium
d) abdominal pain
e) renal stones

3. Patients with malignant hypertension may have:

a) increased risk of pulmonary oedema


b) decreased creatinine clearance
c) proteinuria
d) right bundle branch block

- 35 -
e) pulmonary hypertension

4. In the following conditions, dyspnoea is mainly due to decreased compliance:

a) extensive pulmonary consolidation


b) left ventricular failure
c) massive pulmonary embolus
d) emphysema

5. An increased A-a 02 difference is caused by:

a) decreased functional reserve capacity


b) increased inspired 02
c) N20 absorption
d) hepatic failure
e) Increased V/Q mismatch

6. There is no heart rate response to a Valsalva manoeuvre in:

a) aortic incompetence
b) patients on beta blockers
c) autonomic neuropathy
d) Horner's syndrome
e) increased V/Q mismatch

7. Acute pulmonary oedema can occur with:

a) aortic stenosis
b) tricuspid incompetence
c) myocardial infarction
d) left atrial myxoma
e) massive pulmonary embolus

8. Tension pneumothorax is associated with:

a) mediastinal displacement away from the lesion


b) decreased percussion note on the side of the lesion
c) increased blood pressure
d) stridor
e) cyanosis

9. Atracurium:

a) may cause profound bradycardia


b) activity varies with pH
c) commonly causes histamine release when used in clinical doses
d) causes vagal blockade
e) must not be used in liver failure

10. The laryngeal mask airway:

- 36 -
a) ensures an unobstructed airway
b) is safe to use in patients with a full stomach
c) can be used with positive pressure ventilation
d) can be inserted easily after induction with thiopentone
e) is not associated with laryngospasm

11. Vaporisers-inside-circuit:

a) are efficient
b) are temperature compensated
c) must be placed in the inspiratory limb of the circle
d) have a small volume
e) have a low resistance

12. In a pregnant woman at term:

a) tidal volume is increased


b) functional residual capacity is increased
c) physiological dead space is decreased
d) total vital capacity is reduced
e) airway resistance is reduced.

13. In labour:

a) entonox decreases placental blood flow


b) bupivacaine crosses the placenta freely
c) prilocaine causes methaemoglobinaemia in the foetus
d) epidural analgesia increases delay in second stage

14. The obstetrician calls you for a stuck second twin diagnosed following
administration of ergometrine. The following drugs are capable of relaxing the
uterus:

a) halothane
b) thiopentone
c) suxamethonium
d) salbutamol
e) ritodrine

15. Suxamethonium significantly increases serum potassium concentrations in:

a) quadriplegia
b) myasthenic syndrome
c) malignant hyperpyrexia syndrome
d) adrenocortical insufficiency
e) tetanus

16. Regarding the Bain circuit:

- 37 -
a) inspiratory gas passes through the outer tube
b) it can be used in small children
c) when the inner tube is disconnected, a large increase in dead space is inevitable

17. Diazoxide is used in the treatment of pre-eclampsia because:

a) it causes a diuresis
b) given intravenously it cause hypotension
c) it prevents hypoglycaemia
d) it increases uterine contractility
e) it provides useful sedation

18. Concerning the epidural space:

a) it commences at the foramen magnum


b) it is triangular in shape
c) in the lumbar region, it is widest posteriorly
d) no more than 20 ml of 0.5% bupivacaine should be used for a caesarean section
e) for a caesarean section, the block should extend from T11 to S1

19. Complications of stellate ganglion block include:

a) pneumothorax
b) dural puncture
c) injection into the vertebral artery
d) damage to the phrenic nerve
e) damage to the vagus nerve

20. Renal failure is associated with:

a) normochromic normocytic anaemia


b) acidosis
c) hypernatraemia

21. The caudal space:

a) allows entry into the epidural space


b) contains the spinal cord
c) is covered by the sacrococcygeal membrane

22. Concerning automatic blood pressure devices:

a) they over-read at high systolic blood pressure


b) they over-read at low systolic blood pressure
c) they can cause ulnar nerve damage
d) they are affected by arrhythmias
e) the width of the cuff should be half the circumference of the arm
f) they can cause ulnar nerve palsy

23. Concerning the Severinghaus electrode:

- 38 -
a) sodium bicarbonate is the electrolyte solution
b) it contains CO2 sensitive glass
c) it is affected by temperature
d) it is affected by nitrous oxide
e) it is more accurate with blood than with gases

24. Helium:

a) is less viscous than oxygen


b) alters the voice
c) can be used in the treatment of bronchospasm
d) supports combustion
e) is a liquid stored in brown cylinders

25. Recognised causes of urinary retention include:

a) ketamine
b) morphine
c) amitryptyline
d) ephedrine
e) frusemide

26. Regarding pulmonary artery catheters in a normal person:

a) the wedge pressure is about 12 mmHg


b) the pulmonary artery pressure is about 20/5 mmHg
c) the central venous pressure is about 5 cm H2O
d) the right ventricular pressure is about 30/0 mmHg
e) the internal jugular to wedged distance is about 70 cm

27. Regarding brain stem death:

a) a diagnosis can be made during a convulsion


b) criteria cannot be met in the presence of spinal reflexes
c) an EEG is required for the diagnosis
d) glucose has to be normal

28. Typical total parenteral nutrition requirements for a 70 kg man include:

a) 14 g of nitrogen
b) glucose
c) magnesium
d) 1 ml of water for 1 kcal of energy
e) no intralipid in hepatic failure

29. Regarding intermittent mandatory ventilation:

a) it must have a mandatory breath rate of 8-10 bpm


b) it should be used with positive end-expiratory pressure

- 39 -
c) all breaths are triggered by the patient
d) it is synchronised to the patient's expiration
e) it gives an abnormal end tidal CO2 result

30. A pressure cycled ventilator:

a) is a minute volume divider


b) is often cycled from expiration to inspiration by time
c) is cycled from inspiration to expiration after a set time
d) has end-expiratory pressure determined by the patient's compliance
e) ventilates a patient more efficiently with obstructive airways disease if operated at
low flows

31. Concerning exponential decay:

a) ½ time constant equals a half life


b) 3 x time constant equals 97%
c) the rate of change is proportional to the quantity at a certain time
d) at one time constant, 37% remains
e) the time constant is the time at which the process would have been complete, had
the initial rate of change continued

ANSWERS
 
1.TTTF 24.FTFFF
2.TTFTT 25.FTTTF
3.TTTTT 26.TFTFF
4.TTFF 27.FFFT
5.TTTTT 28.TTTTF
6.FTTFF 29.FFFFF
7.FFTTF 30.FTTFF
8.TFFFT 31.FTTTT
9.TTTFF
10.FFTFF  
11.FFFTT
12.TFTFF
13.?TTT
14.TTFTT
15.TFTFT
16.FFT
17.FTTFF
18.TTFFF
19.TTTTT
20.TTF
21.TFT
22.FTTTTT
23.TFTFT
 

MCQ G
Created: 6/7/2004
  1. The following increase intraocular pressure in the normal eye:

- 40 -
a) hypercarbia
b) acetazolamide
c) atropine
d) hypotension
e) respiratory obstruction

2. Intraluminal gastrointestinal pressure is increased by:

a) nitrous oxide anaesthesia


b) epidural block to T6
c) neostigmine
d) morphine
e) adrenaline

3. In the elderly:

a) systolic hypertension is common


b) ventilatory response to CO2 is normal
c) PaO2 is lower than in young adults
d) upper airway reflexes are impaired
e) postoperative analgesic requirements are increased

4. In one lung anaesthesia, PaO2 depends on:

a) the amount of blood flowing through the non-ventilated lung


b) FiO2
c) intraoperative haematocrit
d) cardiac output
e) mixed venous oxygen tension

5. Regarding bupivacaine:

a) it causes cardiac toxicity before convulsions


b) more than 50% is ionised at physiological pH
c) more than 20% is excreted unchanged
d) it is less potent than prilocaine
e) it crosses the placenta more slowly than lidocaine

6. For the same amount of lidocaine (mg), a greater segmental block is achieved:

a) in the pregnant near term


b) in the elderly
c) with higher concentration
d) with rapid injection
e) with addition of 1:200,000 adrenaline

7. Regarding epidural opioids:

a) less than 2-3% morphine crosses into the CSF


b) pethidine crosses into the CSF faster than morphine

- 41 -
c) prior injection of local anaesthetic decreases the amount of morphine that crosses
into the CSF
d) relatively more diamorphine gains access to the systemic circulation than morphine
e) delayed respiratory depression does not occur with the very fat soluble opioids

8. Regarding spinal anaesthesia:

a) the spinal cord terminates at L4


b) barbotage increases the spread of subsequent block
c) aqueous 0.5% bupivacaine is hyperbaric
d) bradycardia may be treated with IV metaraminol
e) high block impairs the ability to cough

9. The management of air embolism during posterior fossa surgery may include:

a) positioning the patient on the right side


b) administration of mannitol
c) raising cerebral venous pressure
d) discontinuation of nitrous oxide
e) rapid infusion of fluid

10. Regional cerebral metabolism is increased by:

a) halothane
b) mannitol
c) pain
d) ketamine
e) sodium thiopentone

11. Regarding the flow-directed multi-lumen PA catheter:

a) the thermistor is situated 20 cm from the tip


b) pulmonary capillary wedge is a small bronchopulmonary segment
c) cardiac estimations can be performed by pressure measurement
d) an open central lumen when balloon is inflated protects against distal infarction
e) it is more accurate when inserted via the internal jugular vein than via the
subclavian

12. Helium:

a) is less viscous than oxygen


b) is stored as a liquid in cylinders
c) is used to decrease the work of breathing in bronchospasm
d) changes the voice
e) supports combustion

13. Regarding ankylosing spondylitis:

a) it is more common in females than males after the age of 40


b) it is associated with bilateral hip arthropathy

- 42 -
c) it is associated with iritis
d) low grade pyrexia may be present
e) may present with sciatica

14. Regarding the trachea:

a) the only blood supply is from the bronchial arteries


b) it starts at C4
c) it ends at T5
d) it is lined by transitional epithelium
e) it is 1.5 - 2 cm wide in the adult

15. Minimum alveolar concentration:

a) is greater in men than women


b) decreases with age
c) is lower in the neonate compared with a 2 year old
d) is lower in pregnancy
e) is decreased with acute alcohol intoxication

16. Spinal versus general anaesthesia for repair of fracture of neck of femur:

a) decreases mortality
b) reduced hospital stay
c) decreases the incidence of thromboembolism
d) provides better immediate postoperative pain relief
e) decreases intraoperative blood loss

17. Regarding the Severinghaus electrode:

a) the electrolyte is sodium bicarbonate


b) it has CO2 sensitive glass
c) it is affected by temperature
d) it is more accurate for blood than gas sample anaylsis
e) it is affected by nitrous oxide

18. In arterial blood gases:

a) the respiratory quotient is affected by parenteral nutrition


b) (A-a) O2 at FiO2 1 is less than 15 kPa normally
c) more than usual amounts of heparin increase pH
d) mixed venous pH is equal to arterial pH
e) dissolved oxygen increases with hypothermia

19. In the anabolic phase after surgery:

a) calories should be supplied in a ratio of 150:1 g nitrogen


b) there is formation of 30 g protein per day
c) potassium excretion increases
d) tissue fats are mobilised

- 43 -
20. ACE inhibitors cause:

a) cough
b) sodium and water retention
c) vasodilation
d) tachycardia

21. In the anterior interscalene approach to the brachial plexus:

a) it is performed at C6 level
b) interscalene groove is accentuated by deep inspiration
c) Horner's syndrome is essential for a successful block
d) total spinal anaesthesia may result
e) pneumothorax is rare

22. In a 2 year old child:

a) the narrowest point of the trachea is the cricoid ring


b) fluid requirements are 100 ml/kg/day
c) blood volume is 50 ml/kg
d) chest wall compliance is decreased compared with the adult
e) there is increased platelet function

23. In a patient with untreated megaloblastic anaemia:

a) histamine-fast achlorhydria is always present


b) urine methyl malonate is increased
c) there is an association with gastric cancer
d) resection of ileal loops for Crohn's disease may be a causative factor
e) serum B12 levels may be normal

24. Prolonged exposure to 50% nitrous oxide for 3 days causes:

a) megaloblastic marrow change


b) peripheral neuropathy
c) lymphopenia
d) decreased methionine synthetase activity
e) increased sensitivity to suxamethonium

25. Glycosuria may occur:

a) in pregnancy
b) in phaeochromocytoma
c) in hypopituitarism
d) in partial gastrectomy
e) after subarachnoid haemorrhage

26. Hypothyroidsm is associated with:

- 44 -
a) supraventricular tachycardia
b) slow relaxation of tendon reflexes
c) low serum cholesterol
d) high serum protein-bound iodine

27. Regarding the radial nerve:

a) it supplies the extensor compartment of the forearm


b) damage is associated with sensory loss over the lateral aspect of the forearm only
c) damage is associated with alcoholism
d) it can be damaged by pressure on the lateral epicondyle of the humerus

28. Tissues which can rapidly regenerate after damage include:

a) hepatocytes
b) renal glomerular cells
c) anterior horn cells
d) aortic smooth muscle
e) epithelial cells at the edge of a peptic ulcer

29. A drug that blocks dopamine receptors only will:

a) be anti-arrhythmic
b) decrease renal perfusion
c) improve gut blood flow
d) improve Parkinson's disease
e) prevent motion sickness

30. Goldman criteria include:

a) previous cardiac surgery


b) mitral valve disease
c) hypertension
d) atrial fibrillation
e) previous myocardial infarction

31. APACHE II utilises:

a) emergency surgery
b) haemoglobin
c) admission after cardiac arrest
d) (A – a) O2

32. Coarctation of the aorta:

a) is usually preductal
b) is associated with cerebral aneurysms
c) may be associated with differential cyanosis
d) may need left heart bypass during correction
e) is repaired via a right thoracotomy

- 45 -
33. During cardiopulmonary resuscitation:

a) lidocaine should be given before adrenaline in ventricular fibrillation


b) DC cardioversion with 50 J is the treatment of choice for ventricular tachycardia
c) adrenaline 0.5 mg should be given via endotracheal tube
d) 50 ml 8.4% bicarbonate should be given every 10 minutes
e) IV calcium is indicated for patients on dialysis

34. Hypercalcaemia can be caused by:

a) vitamin D deficiency
b) vitamin D intoxication
c) decreased serum calcitonin
d) primary hyperparathyroidism
e) oral potassium chelators

35. Alkaline phosphatase is increased in:

a) osteitis deformans (Paget's disease)


b) multiple chondromata
c) hyperparathyroidism
d) prostatic carcinoma in situ
e) liver disease

36. Creatinine phosphokinase:

a) is raised 24 hours after myocardial infarction


b) untreated hyperthyroidism
c) osteomalacia
d) duchenne muscular dystrophy
e) myasthenia gravis

37. Serum sodium 125 mmol/L and serum potassium 6.25 mmol/L can be found
in:

a) renal failure
b) Addison's disease
c) hypoaldosteronism
d) liver tension
e) potassium deficiency

38. Serum urea of 20 mmol/L is consistent with:

a) dehydration
b) A gastrointestinal tract bleed
c) severe hepatic impairment

39. Atrophy of the tongue occurs in:

- 46 -
a) myotonia
b) syringobulbia
c) pseudobulbar palsy
d) Parkinson's disease
e) Multiple sclerosis

40. Metabolic acidosis is present in:

a) renal failure
b) respiratory acidosis
c) ureterocolic anastomosis
d) severe vomiting
e) diarrhoea

41. In the oliguric phase of acute tubular necrosis:

a) small volumes of concentrated urine are produced


b) there is hyperkalaemia
c) there is malignant hypertension
d) there is a progressively rising ventral venous pressure
e) there is a raised serum urea with a normal serum creatinine

ANSWERS

 
1.TFTFT 24.TFTTF
2.TTTFF 25.TTFTT
3.TFTTF 26.FTF?
4.TTFTT 27.TFTF
5.FTFFT 28.TFFFT
6.TTTFF 29.FTFFF
7.?FFTT 30.FFFTT
8.FFFFT 31.TTTTT
9.FFTTT 32.FTFTF
10.FFTTF 33.FTFFT
11.FFTFF 34.FTTTT
12.FFFTF 35.TFTFT
13.FTTTT 36.TFFTF
14.FFTFT 37.TTTTF
15.FTTTT 38.TTF
16.FFTTT 39.FTTFT
17.TFTTF 40.TFTFT
18.TTFFT 41.FTFTF
19.?TTT
20.TFTT
21.TTFTT
22.TTFFF
23.FTTTT
 

MCQ H
Created: 16/2/2005

- 47 -
  42. In severe hepatocellular failure:

a) albumin levels are low


b) fibrinogen levels are low
c) prothrombin time is shortened
d) plasma sodium levels are raised
e) bilirubin is raised

43. In severe hepatic impairment, drug pharmacokinetics are altered by:

a) decreased protein binding


b) change in body fluid compartments
c) competition for protein binding sites with toxins
d) increased lipid solubility of the drug itself
e) decreased albumin levels

44. Regarding gentamicin vestibular toxicity:

a) it is reversible
b) it is more common in the elderly
c) it does not occur after oral ingestion
d) caloric tests are normal
e) it is more likely to occur with liver disease

45. In the diagnosis of brain stem death:

a) consultation with a neurologist is needed


b) an EEG must be flat for 24 hours
c) convulsions preempt the diagnosis
d) spinal reflexes may be present
e) blood must be sent for drug screening

46. Warming blood to 37 degrees C during massive blood transfusion:

a) decreases the risk of citrate toxicity


b) increases plasma potassium concentration
c) increases plasma carbon dioxide tension
d) decreases the incidence of arrhythmia
e) increases CO2 buffering capacity of cells

47. Percutaneous absorption occurs with:

a) atropine
b) hyoscine
c) morphine
d) fentanyl
e) paracetamol

48. Salicylate overdose is associated with:

- 48 -
a) fibrinolysis
b) hypothermia
c) respiratory acidosis
d) polyuric renal failure
e) massive gastric bleeding

49. Non-steroidal analgesic agents:

a) can be given intrathecally


b) cause thrombocytopenia
c) block prostaglandin synthetase
d) cannot be used in gastrointestinal tract disease

50. Regarding stellate ganglion block:

a) it causes deafness
b) it may cause phrenic nerve paralysis
c) oesophageal perforation may occur
d) a vasovagal reaction may occur during its performance
e) if successful, it causes a change in skin resistance on stimulation

51. You are asked to attend to a woman, 30 minutes after delivery, who is fitting.
The causes for this may be:

a) eclampsia
b) local anaesthetic toxicity
c) grand mal epilepsy
d) hyperventilation
e) cerebral thrombosis

52. Malignancy is associated with:

a) familial polyposis coli


b) ulcerative colitis
c) villous papilloma
d) chronic constipation
e) chronic diverticulitis

53. Regarding the measurement of an anaesthetic agent:

a) infrared measurement is agent specific


b) mass spectrometry is not agent specific
c) nitrous oxide interferes with paramagnetic analysers
d) acoustics can be used for measurement
e) infrared analysers pick up isopropyl alcohol

54. Risks of electrocution are decreased by:

a) conductive flooring
b) the use of fuses

- 49 -
c) the use of isolating transformers
d) the use of battery power
e) each part of the apparatus being attached to separate earth sockets

55. Haematuria may occur in:

a) rheumatic fever
b) bacterial endocarditis
c) hydronephrosis
d) prostatic hypertrophy
e) acute glomerulonephritis

56. The following increase lower oesophageal sphincter tone:

a) metoclopramide
b) atropine
c) cisapride
d) domperidone
e) morphine

57. Inhalation of gastric contents will produce:

a) pulmonary oedema
b) bronchoconstriction
c) left ventricular failure
d) hypovolaemia
e) reduced activity of surfactant

58. Muscle relaxant reversal may be difficult in the presence of:

a) hyperkalaemia
b) fentanyl overdose
c) hypermagnesaemia
d) hypocapnia
e) acidosis

59. Extrapyramidal effects can be seen following:

a) chlorpropamide
b) terfenadine
c) metoclopramide
d) domperidone
e) perphenazine

60. The following statements are true:

a) Cerebrospinal fluid (CSF) should be examined if a brain tumour is suspected


b) CSF is characteristically normal in trigeminal neuralgia
c) CSF protein greater than 1.5 g may be found in motor neurone disease
d) CSF protein is raised in untreated meningococcal meningitis

- 50 -
e) CSF cell count is increased in Guillain-Barre syndrome

61. Treatment for epilepsy includes:

a) ethosuximide
b) sodium valproate
c) clonazepam

62. Lung compliance is increased in:

a) the presence of intra-alveolar fluid


b) acute respiratory distress syndrome
c) idiopathic pulmonary fibrosis
d) emphysema
e) fibrosing alveolitis

63. In a pregnant woman at term:

a) tidal volume is increased


b) functional residual capacity is increased
c) physiological dead space is decreased
d) total vital capacity is reduced
e) airway resistance is reduced

64. Suxamethonium significantly increases serum potassium concentration in:

a) quadriplegia
b) myasthenic syndrome
c) malignant hyperpyrexia
d) adrenocortical insufficiency
e) tetanus

65. Coeliac plexus block:

a) can be used to relieve pain from intra-abdominal malignancy


b) may cause orthostatic hypotension
c) may cause diarrhoea
d) can be used to treat pain in acute pancreatitis
e) causes constriction of the sphincter of Oddi

66. Breathing 10% oxygen in nitrogen in a fit individual causes:

a) central cyanosis
b) increased cardiac output
c) reduced stroke volume
d) reduced mixed venous oxygen content
e) angina pain

67. The use of oral antibiotics in the preparation for colonic surgery may cause:

- 51 -
a) fungal infection of the bowel
b) diarrhoea
c) fulminant enterocolitis
d) altered response to muscle relaxants
e) hepatic failure

68. A raised reticulocyte count is found in:

a) untreated pernicious anaemia


b) aplitic anaemia
c) untreated iron deficiency anaemia
d) chronic lymphocytic anaemia
e) congenital spherocytic anaemia

69. Regarding a prolapsed disc at L5 – S1:

a) it is associated withscoliosis
b) it is associated with loss of sensation on the medial side of the right calf
c) it is associated with loss of knee jerk
d) onset of urinary incontinence requires urgent surgical treatment
e) a plaster cast is the treatment of choice

70. Ulcerative colitis is associated with:

a) cirrhosis
b) cholangitis
c) clubbing
d) iritis
e) arthritis

71. Neuropraxia:

a) is more common after long operations


b) does not occur with local anaesthetics
c) does not occur with muscle relaxants
d) only occurs when previous neuropathy is present
e) takes a long time to recover

72. Ritodrine can cause:

a) bradycardia
b) heart block
c) left ventricular failure
d) hypotension
e) peripheral vasoconstriction

73. Pressure cycled ventilators:

a) are minute volume dividers


b) are cycled from inspiration to expiration after a set time

- 52 -
c) may be cycled from expiration to inspiration after a set time
d) have an end-expiratory pressure determined by the patient's compliance
e) ventilate a patient with chronic airways obstructive disease better if operated at low
flows
f) are not associated with a risk of barotrauma

74. In the elimination of drugs:

a) alkalinisation of urine enhances elimination of weak acids


b) oxidation increases elimination of polar compounds
c) glucuronidation occurs at the microsomal level
d) terminal half life determines drug dose intervals
e) a decrease in glomerular filtration rate reduces gentamicin excretion

75. In a circle anaesthetic system:

a) the soda lime canister should be equal in volume to the patient's tidal volume
b) the system will function satisfactorily with a single one-way valve
c) if a vaporiser is put in the circle, it should be placed between the inspiratory limb
and the fresh gas flow
d) the volume of the reservoir bag is not critical
e) the adjustable pressure limiting valve should be placed between the expiratory limb and
the soda lime

76. Concerning intermittent positive pressure ventilation and airway pressures:

a) peak inspiratory pressure decreases if there is a leak


b) a pneumothorax increases peak and plateau pressures
c) there is an increase in peak pressure with a decrease in compliance
d) bronchial constriction causes a rise in plateau pressure
e) peak pressures are high in obstructive airway disease

77. Concerning intra-arterial pressure transducers:

a) the natural resonant frequency may be determined by square waves at 25 Hz via the
intraflow system
b) they are best used with long narrow catheters
c) a resonant frequency of 10 Hz is ideal
d) mean pressure is not affected by damping
e) optimum damping occurs at 5 times the resonant frequency

78. The following may attenuate the pressor response to intubation:

a) ACE inhibitors
b) calcium antagonists
c) intravenous lidocaine
d) a large dose of induction agent
e) beta-blockers

79. Regarding syncope:

- 53 -
a) the most common cause is cardiovascular disease
b) there may be an association with bradycardia
c) it may be preceded by a loss of vision
d) it is due to a fall in cardiac output
e) it may be indirectly caused by urinary retention

80. Diplopia may occur in:

a) myasthenia gravis
b) retrobulbar neuritis
c) cerebellar hemisphere disease
d) Horner's syndrome

81. A Wright's respirometer:

a) is essentially a turbine
b) is affected by viscosity
c) is temperature compensated
d) is accurate at flows of <1 litre per minute
e) over-reads at high flows

82. In the management of the amitryptiline overdose, the following may be


useful:

a) digitalisation
b) beta-blockers
c) alkaline diuresis
d) atropine
e) isoprenaline infusion

83. Infant respiratory distress syndrome:

a) usually occurs within 12 hours of birth


b) is more common after caesarean section
c) has a better prognosis if steroids are given to the infant
d) is uncommon after 36 weeks' gestation
e) is more common in multiple pregnancies

84. The coeliac ganglion is:

a) in front of L3
b) on the crura
c) behind the pancreas
d) behind the inferior vena cava
e) in front of the aorta

85. Factors leading to hypothermia include:

a) vasodilation

- 54 -
b) exposure of abdominal contents
c) neuromuscular blockers
d) spinal anaesthesia
e) dry gases

86. In endotracheal tubes:

a) pressure is inversely related to length in laminar flow


b) irregularities within the tube increase resistance
c) at angled connectors, turbulence increases as gas flow rate increases
d) in laminar flow, the pressure drop along the tube is linearly related to length

87. TENS:

a) works via A fibres


b) is useful for peripheral nerve injuries
c) is useful for thalamic pain
d) uses voltages of 0 to 50 volts
e) uses frequencies of 1 to 100 Hz

88. Intravenous guanethidine:

a) is useful in sympathetic dystrophy


b) works on the parasympathetic system
c) can be repeated
d) requires the tourniquet to be applied for an hour
e) causes hypotension

ANSWERS

 
42.TTFFT 61.TTT 81.TFFFT
43.TTT?T 62.FFFTF 82.FTTFF
44.FTTFF 63.TFTFF 83.FFFTT
45.FFTTF 64.TFTFT 84.FTTTT
46.TFFTT 65.TTTTF 85.TTTTT
47.FTFTF 66.TFTTF 86.FTTT
48.FFFFF 67.?T?TF 87.TTFFT
49.FTTT 68.FFFFT 88.TFTFT
50.FTTTT 69.FFFTF
51.TTTFF 70.TTTTT
52.TTTFF 71.TFFFF
53.FFFTT 72.FFTTF
54.FTTTF 73.FTTFFF
55.FTFTT 74.TFTTT
56.TFTTF 75.FFTFT
57.TTFTT 76.TTTFT
58.FTTFT 77.TFFTT
59.FFTTT 78.TTTTT
60.FTFTF 79.TTTTT
80.TFFF
 
 

- 55 -
MCQ I
Created: 15/2/2005
1. The following agents may cause pulmonary fibrosis:

a) bleomycin
b) cortisone hemisuccinate
c) beryllium
d) paraquat
e) organophosphate compounds

2. In comparison with an older child, a newborn infant increases pulmonary


ventilation by breathing more often rather than more deeply. This is related to:

a) total respiratory compliance


b) the horizontal disposition of the rib cage
c) immature respiratory chemoreceptors
d) non-respiratory acidosis found in neonates
e) coordination of respiratory effort with sucking movement

3. The most common site of laryngeal granuloma after short-term intubation is:

a) the piriform fossa


b) the epiglottis
c) the anterior 1/3 of the vocal cords
d) the posterior 1/3 of the vocal cords
e) the trachea

4. Regarding measurement of anaesthetic gases:

a) infra-red analysers are specific for agents


b) mass spectrometry may not be specific for agents
c) infra-red analysers pick up isopropyl alcohol
d) acoustic techniques may be used to measure gas concentrations
e) nitrous oxide interferes with paramagnetic analysers

5. Regarding myasthenia gravis:

a) IgE antibodies are found in 85% of patients


b) muscle weakness improves with exercise
c) muscle weakness is worsened by gentamicin
d) plasma exchange produces rapid remission
e) thymectomy is the treatment of choice in patients over 50 years

6. A raised reticulocyte count is found in:

a) untreated pernicious anaemia


b) aplastic anaemia
c) untreated iron deficiency anaemia
d) chronic granlocytic leukaemia
e) congenital spherocytic haemolytic anaemia

- 56 -
7. Measurement of prothrombin time is helpful in:

a) haemophilia
b) von Willebrand's disease
c) scurvy
d) jaundice
e) idiopathic thrombocytopenic purpura

8. Propofol:

a) is suspended in an emulsion of soya bean oil and egg phosphatide


b) can produce green urine
c) has little effect on the cardiovascular system
d) impairs ventilatory response to CO2
e) has no effect on intraocular pressure

9. Essential criteria for the diagnosis of brain stem death are:

a) equal pupils
b) absent doll's head response
c) absent limb movements
d) patient's temperature must exceed 34 degrees C
e) PaCO2 must exceed 6.5 kPa at completion of apnoea testing

10. Malignant hypertension is associated with:

a) increased risk of pulmonary oedema


b) right bundle branch block
c) reduced creatinine clearance
d) proteinuria
e) pulmonary hypertension

11. Coarctation of the aorta:

a) is a congenital condition
b) produces upper limb hypertension
c) may be associated with a displaced apex beat
d) produces a diastolic murmur over the precordium
e) produces skeletal abnormalities on a plain chest X-ray

12 Routine laboratory investigation of a 42-year-old man reveals an elevated


creatinine phosphokinase. This finding may indicate:

a) abnormal metabolism of haemoglobin


b) the patient is at increased risk of malignant hyperpyrexia under anaesthesia
c) hepatocellular dysfunction
d) recent exercise
e) recent myocardial infarction
f) Duchenne muscular dystrophy

- 57 -
g) hypothyroidism

13. Febrile collapse following a colectomy may be due to:

a) pulmonary embolus
b) breakdown of the anastomosis
c) haemorrhage
d) sepsis

14. pH 7.5, PCO2 7kPa, PO2 12 kPa, B.E. +9.5


These arterial blood gas values could represent:

a) pyloric stenosis for several days


b) the administration of 50 mmol of bicarbonate following cardiac arrest
c) excessive deadspace ventilation
d) a severe asthmatic on steroids
e) chronic obstructive airways disease

15. A young man admitted to casualty following a road traffic accident is found
to have central dislocation of the hip and is shocked. Likely causes are:

a) ruptured bladder
b) ruptured urethra
c) blood loss
d) neurogenic shock
e) fat embolism

16. A patient with vomiting, respiratory distress, cyanosis, epigastric tenderness


and subcutaneous emphysema in the neck may be suffering from:

a) ruptured oesophagus
b) ruptured diaphragm
c) ruptured trachea
d) spontaneous pneumothorax
e) pulmonary embolus

17. The following are features of malignant hyperpyrexia:

a) hypoxia
b) hypocapnia
c) hyperkalaemia
d) prolonged neuromuscular blockade

18. In patients with porphyria:

a) griseofulvin may precipitate an acute attack


b) glycine should not be used during transurethral resection of the prostate
c) dysautonomia may occur
d) preoperative fluid restriction is beneficial
e) fentanyl may safely be used

- 58 -
19. In carcinoid syndrome:

a) urine turns pink on standing


b) aortic valve lesions frequently occur
c) there is an association with obstinate constipation
d) diarrhoea and bronchospasm may occur
e) severe flushing occurs
f) ketanserin, octreotide and aprotonin should be given

20. Regarding anaphylactoid reactions:

a) they are clinically indistinguishable from anaphylaxis


b) widespread erythema occurs
c) adrenaline is the treatment of choice
d) they are not commonly caused by etomidate

21. For a patient suffering from Parkinson's disease on L-dopa, the following
agents should not be used:

a) enflurane
b) droperidol
c) nitrous oxide
d) morphine
e) fentanyl

22. During cardiopulmonary resuscitation:

a) lidocaine should be used before adrenaline in ventricular fibrillation


b) calcium is appropriate in patients undergoing haemodialysis
c) cardioversion at 50 J is indicated for ventricular tachycardia
d) adrenaline 0.5 mg is an appropriate dose for administration via an endotracheal
tube
e) bicarbonate should be administered every 10 minutes

23. The risk of electrocution may be reduced by:

a) conductive flooring
b) the use of isolating transformers
c) a separate earth for each socket
d) the use of fuses
e) battery powered equipment

24. Ventricular arrhythmias are more common in the presence of:

a) hypokalaemia
b) hypoxia
c) thyrotoxicosis
d) cardiopulmonary bypass and digoxin treatment
e) essential hypertension

- 59 -
25. Hypoglycaemic coma:

a) occurs with hypothermia


b) may cause coma in adrenal failure
c) is a recognised complication of untreated thyrotoxicosis
d) occurs in normal people after 48 hours of fasting

26. In arterial blood gases:

a) heparin in significant amounts will cause an elevated pH


b) there is an increase in dissolved oxygen with hypothermia
c) when the FiO2 is 1 the alveolar-arterial pressure difference exceeds 15 kPa

27. An adult breathing 100% oxygen at sea level may suffer from:

a) retrosternal chest pain


b) convulsions
c) dizziness
d) atelectasis
e) permanent visual damage

28. In the measurement of cardiac output using a thermistor:

a) the temperature measurement is accurate to 0.1 degree C


b) the thermistor is proximal to the balloon
c) results are inaccurate after the catheter has been in use for more than 48 hours
d) the thermistor measures the true core temperature

29. The following are associated with ulcerative colitis:

a) iritis
b) cirrhosis
c) cholangitis
d) arthropathy
e) finger clubbing

30. Features of ankylosing spondylitis include:

a) it is more common over the age of 40 and in males rather than females
b) low grade pyrexia
c) sciatica
d) hip arthropathy
e) iritis

31. In L5-S1 disc prolapse with sciatica in the right leg:

a) loss of the knee jerk on the right occurs


b) loss of sensation in the medial right calf occurs
c) incontinence requires further surgical investigation

- 60 -
d) plaster of Paris cast is the preferred early treatment
e) it is commonly associated with scoliosis

32. Cauda equine syndrome is more common in:

a) old age
b) use of epidural adrenaline
c) lithotomy position
d) cases where barbotage is used
e) face down position

33. Suitable anaesthetic techniques for patients with raised intracranial pressure
are:

a) nitrous oxide, oxygen and fentanyl; controlled ventilation


b) nitrous oxide, oxygen, thiopentone and atracurium
c) ketamine
d) halothane, nitrous oxide and oxygen; spontaneous respiration
e) pre-medication with morphine

34. Epidural opiates may cause:

a) nausea and vomiting


b) motor block
c) urinary retention
d) hypotension
e) respiratory depression

35. An elevated left hemidiaphragm may be caused by:

a) left pneumothorax
b) left pleural effusion
c) situs inversus
d) left phrenic nerve paralysis
e) left lower lobe collapse

36. In formulating total parenteral nutrition for a 30-year-old, 70 kg man


recovering from major trauma:

a) insulin is not needed if less than ? of glucose per day is used


b) 6000 kcal per day would be appropriate
c) there is a risk of sepsis
d) 0.5 mmol of phosphate is needed daily

37. In patients with permanent pacemakers:

a) suxamethonium should be avoided if possible


b) prophylactic antibiotics should not be used
c) diathermy should preferably not be used
d) electrolytes should be corrected prior to anaesthesia

- 61 -
e) anaesthesia can lead to profound hypotension

38. Cricoid pressure:

a) is effective in the presence of a nasogastric tube


b) requires a complete cricoid cartilage to be effective
c) should be performed with the neck extended
d) should be performed after 5 minutes' pre-oxygenation
e) compresses the oesophagus against the cervical vertebrae

39. Regarding EMLA cream:

a) it contains 5% prilocaine and lidocaine


b) it contains adrenaline
c) the melting point of the agents is decreased
d) it can be made with carbonated salts
e) it can be made from any local anaesthetics

40. Regarding caudal anaesthesia:

a) the posterior superior iliac crest can be used as a landmark


b) absorption of local anaesthetic is greater than from lumbar epidurals
c) the dura ends at the level of the posterior superior iliac spine
d) subcutaneous injection can be detected using air
e) negative pressure can be used to detect the space

41. The first rib:

a) is crossed by the subclavian artery


b) is crossed by the vagus nerve
c) is crossed by the supratentorial fascia
d) lies above the stellate ganglion
e) is a landmark for supraclavicular brachial plexus block

42. Regarding the femoral nerve:

a) it arises from the L2, -3 and -4 spinal segments


b) it lies lateral to the femoral artery at the inguinal ligament
c) it supplies the lateral aspect of the thigh
d) total block allows arthroscopy of the knee to be performed
e) block relieves pain from a fractured femur

43. Isoflurane:

a) causes a dose-related decrease in systematic vascular resistance


b) causes respiratory depression
c) is metabolised to inorganic fluoride
d) can cause a tachycardia

44. Regarding intravenous guanethidine:

- 62 -
a) it blocks parasympathetic nerves
b) it cannot be repeated
c) the tourniquet must remain inflated for at least 1 hour
d) it is used as a treatment for sympathetic dystrophy
e) both postural hypotension and diarrhoea are common
f) it causes hypotension

45. The following are true of alfentanil:

a) it is more potent than sufentanil


b) the volume of distribution is greater than that for fentanyl
c) it is more protein bound than fentanyl
d) it has a shorter elimination half-life than fentanyl
e) it is more ionised than fentanyl

46. Pethidine is used in obstetrics because:

a) it is 75% excreted unchanged


b) it does not cross the placenta
c) it causes less respiratory depression than an equipotent dose of morphine
d) it can be used without medical supervision
e) it does not affect uterine contractility

47. In the management of uterine haemorrhage, the following are used:

a) IV fibrinogen
b) IV ergometrine
c) aortic compression
d) uterine packing
e) bimanual compression

48. Ritodrine causes:

a) heart block
b) bradycardia
c) pulmonary oedema
d) peripheral vasoconstriction
e) increased uterine contractility

49. Concerning arterial pressure transducers:

a) mean pressure is not affected by damping


b) critical damping of 0.8
c) an intraflow square wave measures damping
d) optimum damping occurs at 5x the resonant frequency
e) they are best used with long, narrow catheters

50. Causes of respiratory distress in the neonate include:

- 63 -
a) unilateral choanal atresia
b) TDF
c) diaphragmatic hernia
d) necrotising enterocolitis
e) myelomeningocoele

51. Neonates compared with adults have:

a) decreased oxygen consumption


b) decreased ability to shiver
c) increased Vd/Vt
d) increased body surface area/weight ratio
e) increased airway resistance

52. Concerning the use of thiopentone in lower segment caesarean section (LSCS):

a) there is a greater concentration in foetal liver compared with maternal liver


b) the greatest fetal concentration occurs within 2 minutes of maternal administration
c) less crosses the placenta if given at the beginning of a contraction
d) neonatal depression is proportional to peak maternal concentration
e) thiopentone crosses the fetal blood-brain barrier more readily

53. The following are causes of APGAR (<5) following LSCS:

a) maternal hypoxia
b) uterine incision-delivery time greater than 90 s
c) placental transfer of muscle relaxant
d) reduced uterine contractility
e) reduced placental blood flow

54. Suxamethonium does not cross the placenta because of:

a) placental cholinesterase
b) high protein binding
c) elongated molecule
d) high degree of ionisation
e) insufficient maternal concentration

55 Postoperative vomiting is:

a) more common in males than in females


b) reduced with preoperative administration of anticholinergics
c) increased if pain is managed with opiates
d) worse if nitrous oxide is used
e) more common in operations on the abdomen than of the lower limbs

56. Cerebral ischaemia during hypotensive anaesthesia:

a) is worse in a steep head-up position


b) occurs with isoflurane

- 64 -
c) is common when the blood pressure cuff is inflated frequently
d) can be detected with EEG monitoring
e) is avoided if normocarbia is maintained

57. In paracetamol overdose:

a) respiratory alkalosis occurs


b) the patient is hypothermic
c) methionine is used in the treatment
d) acetyl cysteine is an antidote
e) coagulopathy is an early complication

58. Alveolar hypoventilation is associated with:

a) metabolic acidosis
b) asthma
c) upper airway obstruction
d) raised intracranial pressure
e) marked ascites

ANSWERS

 
1.TFTTF 24.TTTTT 47.FTTTT
2.TTFFF 25.FTFF 48.FFTFF
3.FFFTF 26.FTT 49.T?TTF
4.FFTTF 27.TFFTF 50.TTTTF
5.FFTTF 28.TTFF 51.FTFTT
6.FFFFT 29.TTTTT 52.F?T?T
7.FFFTF 30.FTTTT 53.TTFFT
8.TTFTF 31.FFTFF 54.FFFTF
9.FFFFT 32.TFFFF 55.FTFTT
10.TTTTF 33.FTFFF 56.TTFTF
11.TTTFT 34.TFTFT 57.FFTTF
12.FTTTTTT 35.TFTTT 58.FTTTT
13.FTFT 36.FFTF
14.TFFFF 37.TFTTT
15.FFTFF 38.FTTFT
16.TFFFF 39.FFTFF
17.TFTF 40.TTTTT
18.TFTFT 41.TFFFT
19.FFFTTT 42.TTFFT
20.TTTT 43.TTTT
21.FTFFF 44.FFFTTT
22.FTTFF 45.FFTTF
23.FTFTT 46.FFFTT
 
 

ArticleDate:20050215
SiteSection: Article

   

- 65 -
MCQ J
Created: 15/2/2005
1. Recognised causes of abnormal movements of the hands include:

a) respiratory failure
b) renal failure
c) chronic alcoholism
d) liver failure
e) depigmentation and sustantia nigra

2. Recognised complications of renal failure are:

a) pulmonary oedema
b) convulsions
c) paresthesia in hands in the absence of anaemia
d) acidosis
e) hypokalaemia

3. Diabetes mellitus may present with:

a) vulvititis
b) change in refraction
c) paresthesia in legs
d) impotence
e) acute abdominal pain

4. Acute cardiac tamponade may cause:

a) progressive bradycardia
b) cold cyanosed peripheries
c) elevated jugular venous pressure (JVP)
d) giant a waves in JVP
e) pulsus paradoxus

5. Causes of acute pulmonary oedema include:

a) aortic stenosis
b) pulmonary emboli
c) atrial myxoma
d) myocardial infarction
e) chlorine inhalation

6. Regarding hyponatraemia:

a) it should be corrected with hypertonic saline


b) it always implies a disturbance in total body water
c) there is abnormal aldosterone secretion
d) it cannot be interpreted without clinical data
e) it is associated with advanced carcinoma of the bronchus

- 66 -
7. DC cardioversion is indicated for:

a) digitalis toxicity
b) premature atrial contractions
c) ventricular tachycardia
d) supraventricular tachycardia
e) premature ventricular contractions

8. Surgical treatment is the treatment of choice for:

a) Hashimoto's thyroiditis
b) retrosternal goitre
c) longstanding thyroid cyst
d) thyrotoxicosis
e) solitary thyroid nodule

9. Correct treatment for ventricular arrhythmias includes:

a) procainamide
b) digoxin
c) lidocaine
d) carotid sinus massage
e) disopyramide

10. There is an increased risk of development of intestinal malignancy in:

a) pernicious anaemia
b) Peutz-Jegher's syndrome
c) familiar polyposis coli
d) ulcerative colitis
e) diverticular disease

11. Phaeochromocytoma is associated with:

a) elevated 3-hydroxy 4-methoxy mandelic acid in urine


b) bilateral tumours in 50% of cases
c) medullary carcinoma of thyroid
d) frequent metastases of liver
e) hyperglycaemia

12. In a patient on clonidine presenting for surgery:

a) clonidine should be given parenterally through to the postoperative period


b) the patient is at risk of pulmonary embolism during anaesthesia
c) the patient should have a noradrenaline infusion during operation
d) the clonidine should be stopped before surgery
e) clonidine should be stopped on the day before surgery

13. Recognised causes of glycosuria are:

- 67 -
a) pregnancy
b) partial gastrectomy
c) phaeochromocytoma
d) chronic renal failure
e) old age
f) head injury, acromegaly

14. Hiatus hernia is associated with:

a) vitamin B12 deficiency


b) iron deficiency anaemia
c) retrosternal pain
d) lower oesophageal fibrosis
e) hiccoughs

15. Features of lower motor neurone disease are:

a) fasciculation
b) spasticity
c) wasting
d) increased reflexes
e) upgoing plantar response

16. Immediately after complete transection of the spinal cord, the following
features may be found below the lesion:

a) loss of motor power, but preservation of limb reflexes


b) urinary incontinence
c) loss of muscle power, but preservation of sensation
d) flaccid paralysis with loss of limb reflexes
e) loss of muscle power, but preservation of muscle joint position sense

17. Regarding acute pancreatitis:

a) abdominal rigidity and guarding are characteristic early features


b) the serum amylase is characteristically not raised for first 12 hours
c) it may cause adult respiratory distress syndrome
d) hypocalcaemia occurs within the first week of the illness
e) hyperglycaemia is common

18. In a case of a Rhesus negative mother with a Rhesus positive foetus, a high
level of anti-D antibody in the mother is associated with:

a) foetal red cells entering maternal circulation


b) antigen only crossing placenta
c) neonatal anaemia
d) neonatal jaundice
e) sensitisation only occurs in first 3 months of pregnancy

19. Indications for an urgent craniotomy in a person with head injury include:

- 68 -
a) sudden appearance of one fixed dilated pupil
b) rapidly deteriorating conscious level
c) cerebrospinal fluid rhinorrhea
d) falling blood pressure and rising pulse
e) generalised convulsions

20. The following are characteristic features of myasthenia gravis:

a) over 90% suffer from diplopia or ptosis at some stage of their illness
b) symptoms are usually symmetrical
c) muscular weakness is made worse by exercise
d) symptoms characteristically remit during pregnancy
e) steroids and azathioprine may be effective therapy

21. Carcinoma of the lung:

a) can cause Cushing's syndrome


b) may present with a peripheral neuropathy
c) can cause cerebellar degeneration without posterior fossa secondaries
d) can produce Horner's syndrome
e) occurs three times more frequently in men than in women

22. Acute pancreatitis:

a) may present with flank staining


b) may be complicated by abscess formation
c) may be precipitated by steroids
d) is treated with steroids
e) can precipitate tetany

23. The following are features of hyperparathyroidism:

a) polydipsia and polyuria


b) bone pain
c) raised urinary calcium
d) tetany
e) peptic ulceration

24. Digitalis toxicity:

a) may be indicated by bradycardia and a prolonged P-R internal


b) may be indicated by a supraventricular tachycardia or ventricular extrasystoles
c) may be reduced by giving calcium salts
d) is usefully treated by a slow infusion of phenytoin
e) is an absolute contraindication to beta-blockade

25. Clubbing may be seen in:

a) Fallot's tetralogy

- 69 -
b) acromegaly
c) ulcerative colitis
d) cirrhosis
e) myxoedema

26. In haemophilia A:

a) factor VII levels are markedly decreased


b) prothrombin and partial thromboplastin times are usually normal
c) adult sufferers are usually Australian Antibody (HB3Ab) positive
d) drug abuse is a frequent problem
e) the dose of cryoprecipitate to be given prior to surgery is calculated from the
patient's factor VIII level and estimated plasma volume

27. The treatment of adult respiratory distress syndrome should include:

a) steroids
b) prophylactic antibiotics
c) high concentrations of oxygen
d) heparin
e) positive end-expiratory pressure

28. Which of the following are true of hepatitis B (serum hepatitis):

a) it is spread only by blood and blood products


b) most cardiac surgeons acquire the infection
c) it is common in prostitutes, homosexuals and the tattooed
d) it may be prevented by vaccination
e) it remains the commonest cause of post-transfusion hepatitis

29. Acute viral bronchitis and pneumonitis:

a) characteristically affects middle-aged individuals


b) are typically secondary to another infection
c) may be associated with the formation of epithelial giant cells in the alveoli
d) often show hyaline membranes in the respiratory passages
e) may be complicated by the development of lung abscesses

30. Finger clubbing is a recognised feature of:

a) asbestosis
b) chronic alveolitis
c) fibrosing alveolitis
d) fibrocaseous tuberculosis
e) empyema

31. The following features are characteristic of pure mitral stenosis with atrial
fibrillation:

a) a loud first heart sound

- 70 -
b) an apical pre-systolic murmur
c) left ventricular hypertrophy on ECG
d) an elevated left atrial pressure on cardiac catherisation
e) pulsus paradoxus

32. A collapsing pulse is a recognised feature of patients with:

a) malignant hypertension
b) Paget's disease
c) patent ductus arteriosus
d) alcoholic cardiomyopathy
e) atrial septal defect

33. Methaemoglobinaemia may be precipitated by:

a) nitrites
b) methylene blue overdosage
c) aniline dyes
d) ascorbic acid
e) sulphonamides

34. The normal ventricular myocardium:

a) has the ability to contract regularly after complete interruption of the conducting
system
b) has an exceptionally high mitochondrial content
c) responds to an increased workload by hypertrophy
d) responds to an increased workload by hyperplasia
e) is of greater thickness in the right ventricle than in the left ventricle

35. The oxygen content of arterial blood is reduced:

a) by the presence of a left to right shunt in the heart


b) in patients with fibrosing alveolitis when the PaCO2 is low
c) in carbon monoxide poisoning
d) in methaemoglobinaemia
e) in Fallot's tetralogy

36. The following are recognised features of deep vein thrombosis:

a) venography is the single most useful diagnostic test


b) it occurs very commonly in patients who have a fractured neck of femur
c) the initial treatment of choice is warfarin
d) surgical intervention using an umbrella filter is almost always indicated if
symptoms fail to resolve
e) it is often found in association with carcinoma of the pancreas

37. The following features are commonly found in peripheral vascular disease:

a) angina pectoris

- 71 -
b) a normal cholesterol level
c) occlusion at the femoral-popliteal junction
d) it is more common in people from Europe and North America
e) surgery using synthetic prosthesis offers a low occlusion rate

38. Superior vena cava obstruction is commonly characterised by:

a) cyanosis and swelling of the head and arm


b) dyspnoea
c) clubbing
d) Horner's disease
e) nausea and vomiting

39. Radiological findings of a large pulmonary artery on a chest X-ray are


suggestive of:

a) infundibular pulmonary stenosis


b) atrial septal defect
c) Fallot's tetralogy
d) pulmonary valve stenosis
e) Eisenmenger syndrome

40. The following statements are true or false:

a) starvation is characterised by loss of lean muscle mass


b) low zinc levels cause insulin resistance
c) L-amino acids alone are required for protein replacement
d) a normal adult requires 40-50 g of protein/24 hours
e) glucagon secretion is the prime response to trauma

41. Recognised unwanted effects of propranolol include:

a) bronchospasm
b) congestive cardiac failure
c) retinal detachment
d) tachycardia
e) hyperglycaemia

42. In hypothermia:

a) bradycardia is not marked


b) total oxygen consumption is closely related to shivering
c) a zone of dysrhythmia occurs between 26 degrees C and 28 degrees C
d) the dissolved oxygen in plasma is reduced
e) insulin excretion is reduced below 23 degrees C

43. Bleeding from oesophageal varices due to portal hypertension can be


managed medically by:

a) an infusion of 20 units of Pitressin

- 72 -
b) propranolol
c) ice-cold saline infusions into the oesophagus
d) intraperitoneal aspiration
e) a Sengstaken tube

44. A patient with a parathyroid tumour has a calcium level of 4 mmol/L and is
admitted to hospital. Preoperative investigations and management should
include:

a) an intravenous pyelogram
b) an infusion of normal saline
c) administration of magnesium sulphate 10%
d) a skull X-ray and Towne view
e) vitamin D

45. A 40-year-old man is admitted after haemorrhage, dyspnoea and collapse.


He is vigorously resuscitated but somewhat oliguric, and on passage of a
pulmonary catheter the following results were found. RV – 85/15; PA 80/30
mmHg; Wedge pressure 39 mmHg, pulse 110 bpm, systolic pressure 100 mmHg.
As part of his management, this patient requires the following:

a) doxapram
b) dobutamine
c) propranolol
d) spironolactone
e) nitroglycerine

46. Acute tubular necrosis is diagnosed by finding:

a) casts in the urine


b) a urine osmolality exceeding 500 mosm/L
c) a urine specific gravity of 1020
d) a urinary sodium of 10 mmol/L
e) a urinary volume of 700 mol/day

47. You are asked to see a patient on the ward. He has a swinging temperature,
and his sputum and chest X-ray show a right-sided collapse of part of the lung.
Treatment should include:

a) pleural aspiration
b) bronchial lavage
c) surgical biopsy of the appropriate part of the lung
d) bronchography
e) postural drainage

48. In severe airways obstruction:

a) pulsus paradoxus is a useful prognostic guide


b) helium is of benefit
c) the FEV1/FVC ratio is increased

- 73 -
d) alveolar hypoventilation is common
e) the drug of choice is aminophylline

49. A 50-year-old man is admitted with features of obstructive jaundice. The


following investigations are indicated to establish the diagnosis:

a) barium meal
b) IgM levels
c) mitochondrial antibody
d) fibreoptic endoscopy
e) serum alkaline phosphatase

50. An elderly lady who has been on prolonged diuretic therapy presents with
weakness, thirst and painful joints. The following investigations are required:

a) urinary electrolytes
b) serum phosphate
c) an ECG
d) a glucose tolerance test
e) blood gases

ANSWERS

 
1.TFTTT 24.TTFTF 47.FFFFT
2.TTFTF 25.TFTTF 48.FFFFF
3.TTTTT 26.FFTTT 49.TFFTT
4.FTTFT 27.FFTFT 50.FFTTF
5.FFTTT 28.FFTTF
6.FFFTT 29.FFTTF
7.FFTTF 30.TFTTT
8.FTFFT 31.TFFTF
9.TFTFT 32.FTTFF
10.FTTTF 33.TFTFT
11.TFTFT 34.TTTFF
12.TFF?F 35.FTFTT
13.TFTFFT 36.TTFFT
14.FTTTF 37.TFTTF
15.TFTFF 38.TTFFF
16.FTFTF 39.FTFFT
17.TFTTT 40.TFTFF
18.TFTTF 41.TTFFF
19.TFTFF 42.FTFFT
20.TTTFT 43.TFFFT
21.TTTTT 44.TTFFF
22.TTTFT 45.FTFFT
23.TTTFT 46.TFFFF
   

ArticleDate:20050215
SiteSection: Article

cle ti

- 74 -
MCQ K
Created: 15/2/2005
  51. A middle-aged man presents with painless rectal haemorrhage. The
following is indicated:

a) barium meal
b) fibreoptic colonoscopy
c) a prothrombin time
d) an intravenous pyelogram
e) blood gas analysis

52. A patient has the following electrolyte results: Na+ 127 mEq/L, K+6.7 mEq/L.
This could be caused by:

a) oliguric renal failure


b) hyperaldosteronism
c) pituitary insufficiency
d) Cushing's disease
e) Meig's syndrome

53. In a patient with thyrotoxicosis:

a) atrial fibrillation is common


b) diabetes insipidus is a common complication
c) a hoarse voice is not uncommon
d) premedication with pethidine 50 mg, atropine 0.6 mg is the method of choice
e) hypertensive heart disease is a common association

54. In Crohn's disease:

a) the risk of subsequent malignant change is greater than in ulcerative colitis


b) anal lesions are infrequent
c) folate disturbances commonly cause megaloblastic anaemia
d) controlled trials have demonstrated steroids to be the medical treatment of choice
e) the eye signs are indistinguishable from ulcerative colitis

55. An increase in pulmonary vascular resistance is seen in:

a) shock
b) response to vasodilator therapy
c) acidosis
d) head injury
e) hypocapnia

56. Pruritus is seen in association with:

a) miconazole
b) uraemia
c) morphine overdose
d) scabies

- 75 -
e) pancreatitis

57. Carbon dioxide narcosis is associated with:

a) papilloedema
b) raised jugular venous pressure
c) thready pulse
d) cool periphery
e) muscle twitching

58. After major abdominal surgery (e.g. abdomino-perineal resection), the


following metabolic changes would be expected to occur in the first 4
postoperative days:

a) an increased utilisation of glucose


b) a reduction in the amount of sodium excreted in the urine
c) a decrease in the free fatty acid concentration
d) an increase in the amount of potassium in the urine
e) an increase in the oxygen consumption per kg body weight

59. Jaundice due primarily to intrahepatic biliary obstruction (cholestatic


jaundice) is a recognised complication of treatment with:

a) monamine oxidase inhibitors


b) C17-substituted testosterones
c) erythromycin estolate (Ilosone)
d) phenothiazines
e) alkylating agents

60. In Crohn's disease:

a) the risk of subsequent malignant change of the mucosa is greater than in ulcerative
colitis
b) anal tensions are infrequent
c) the demonstration of vitamin B12 malabsorption indicates involvement of the
jejunum
d) eye signs indistinguishable from those complicating ulcerative colitis are found
e) controlled trials have found steroids to be helpful

61. Complications from using a central venous pressure (CVP) catheter for CVP
measurement include:

a) a high incidence of thrombosis and phlebitis


b) a high incidence of predominantly fungal infections
c) abnormal readings of CVP
d) myocardial perforation of pericardial effusion
e) pneumothorax, if the subclavian vein is used

62. Carcinoma of the large intestine:

- 76 -
a) mostly originates in the ascending colon
b) mostly develops from a single polyp
c) may show signet ring features histologically
d) characteristically metastasises to the liver before the lymph nodes
e) is amenable to chemotherapy

63. Finger clubbing is a recognised feature of:

a) asbestosis
b) chronic bronchitis
c) fibrosing alveolitis
d) fibrocaseous tuberculosis
e) empyema

64. Carcinoma of the female breast:

a) is less common than carcinoma of the cervix


b) is unrelated to cystic hyperplasia of the breast
c) when intraductal in type may be associated with Paget's disease of the nipple
d) carries a worse prognosis when auxiliary lymph nodes show proliferation of
histiocytes and plasma cells
e) is prone to metastasis to the internal mammary lymph nodes

65. The following are characteristic of adrenal insufficiency:

a) hypotension
b) small heart
c) hyperkalaemia
d) increased fatiguability of striated muscle
e) hypoglycaemia

66. Regarding blood transfusions:

a) they are associated with an incidence of viral hepatitis of 1% or lower


b) an incompatible transfusion will cause a fall in fibrinogen
c) 1 unit of blood will raise the Hb value by 10% in 24 hours
d) they may result in pulmonary oedema
e) they are responsible for most cases of hepatitis C

67. Hypokalaemia is recognised as a complication of:

a) Zollinger-Ellison syndrome
b) small bowel fistula
c) treatment with bumetanide
d) Cushing's syndrome
e) infestation with round worms

68. Paroxysmal atrial tachycardia can be controlled by:

a) carotid sinus pressure

- 77 -
b) beta-blocking drugs
c) atropine
d) intravenous bolus of lidocaine
e) stellate ganglion blockade

69. Radiologically, a large pulmonary artery on a chest X-ray is associated with:

a) infundibular pulmonary stenosis


b) atrial septal defect
c) Fallot's tetralogy
d) pulmonary valve stenosis
e) Eisenmenger syndrome

70. A man of 50 years of age presents with a left-sided pneumonia. The following
is indicated:

a) doxapram infusion at 2 mg/kg/h


b) blood culture
c) bronchoscopy
d) hydrocortisone infusion
e) physiotherapy with the Bird ventilator

71. A patient undergoing anaesthesia develops a blood pressure of 240/140


mmHg during the procedure. The initial treatment should include:

a) digitalisation
b) urinary vanillyl mandelic acid levels
c) IV guanethidine
d) diazoxide
e) phenoxybenzamine

72. A woman, aged 30 years, who has suffered from asthma suddenly becomes
increasingly breathless. The following investigations are particularly useful:

a) ECG
b) peak expiratory flow
c) eosinophil count
d) serum glucose
e) clinical examination of sputum

73. Calcification on the chest X-ray in a single lesion is a feature of:

a) bronchial carcinoma

ANSWERS

 
51.FTFFF 63.FFTTT
52.TFTFT 64.FFTFT
53.TFFFF 65.TFTTT

- 78 -
54.FFFFT 66.TTTTT
55.TFTTF 67.TTTTF
56.TTFTF 68.TTFFF
57.TTFFT 69.FTFFT
58.FTFTT 70.FTTFF
59.FTTTF 71.FFFTT
60.FFFTF 72.TTFFT
61.TFTTT 73.F
62.FFTFF

ArticleDate:20050215
SiteSection: Article on: Article

- 79 -

You might also like